Outline of Evidence

Note: this entry is based in United States law

I. Burdens and Presumptions

a. Burden of Proof

i. Burden of pleading

1. You have to plead a claim, to state a claim for which relief can be granted

ii. Burden of production

1. Must introduce evidence to support your claim in your pleading

iii. Burden of persuasion

1. Must convince the jury

a. Civil case: preponderance of evidence

b. Criminal case: beyond reasonable doubt

c. (I think I got those right)

b. Presumptions

i. If you prove a preliminary set of facts, the jury will be instructed to find some other fact that’s hard to prove directly-absent proof to the contrary.

ii. Examples: mailed letter, assume it was received; person hasn’t been seen for 7 years, presumed dead.

iii. All a presumption does is shift the burden of production on that particular issue.

iv. Irrebuttable presumptions are really just substantive rules of law; they don’t shift the burden of production, they just set forth the law.

II. Relevance (401, 402, 403)

a. How do you determine if something is relevant?

i. Makes more or less probable (Probative)

ii. Any fact of consequence (Material)

b. The evidence must relate to a material fact and must make that fact more or less probable.

c. Materiality: US v. James

i. Mother(D) charged with manslaughter for giving her daughter gun that daughter used to shoot mother’s BF

ii. Held -Having a criminal record makes it more likely that V bragged about the events. (It is probative of a material fact-his record is probative of his having told her stories about past acts, which stories put her in fear for safety.)

iii. Note: evidence of past crimes went to Mom’s credibility (in saying that he’d told her about these things). Credibility is always a material issue.

iv. This is also an example of conditional relevance-his criminal history is only relevant if the Mom knew about it.

1. [note: although it was relevant, it was barred under 403 because it might unduly prejudice the jury into thinking he was a bad man if they knew his criminal record. Substantially outweighed its probative value.]

d. Old Chief:

i. Can limit information about crimes to the fact that there WAS one (i.e. D was previously convicted of a felony) and the not the specifics of what the crime was. This helps balance probativeness with risk of harm to D.

e. Conditional relevance (104(b))

i. When the relevance of evidence depends on whether some other fact exists, proof must be introduced for that fact. That proof can be introduced later.

ii. Judge can determine this based on a standard lower than preponderance of the evidence.

iii. Cox case-guy shot through bedroom window. Mom being at the bond reduction hearing was a link in the chain of inferences to the D having a motive to shoot the man; if he didn’t know about the bond reduction hearing (he presumably found out through the Mom) then the bond reduction hearing wouldn’t be probative.

iv. Proof must be sufficient to support a finding that the fact exists. The condition doesn’t really have to be proven (don’t have to prove that Cox knew), just that there’s sufficient proof to reasonably find that he knew.

v. When it comes to conditionally relevant evidence of prior act, the judge must decide whether there is sufficient evidence for the jury to reasonably decide that the conditional fact is true (Huddleston-stolen videotapes in trailer case)

vi. Be sure to know this for exam.

vii. Example: the pen thing. Whether it was relevant depends on whether James knew about the pen thing; if James didn’t know about the pen thing, then it wouldn’t be probative of her fear of him, so it wouldn’t be relevant.

f. Evidence of flight

i. US v. Myers (5th 1977)

1. Held can’t give jury a “flight instruction” that they can infer guilt from his flight where (1) there isn’t clear evidence that he was fleeing (2) if he was, there are other crimes/reasons he might be fleeing the police

ii. Inferences in flight instruction: prove D fled à consciousness of guilt (fled b/c he knew he was guilty) à consciousness of guilt for the crime charged (thought he was guilty, specifically guilty of this crime) à actual guilt

1. In Myers missing guilt of this crime (and in one instance proof that he fled)

g. 403: balancing test for probative evidence

i. If you make it through all the other rules (below), 403 is your last chance to try to keep the evidence out. Always go back to 403 after analyzing under the other rules (and deciding it’s admissible)! Also consider requesting a 105 limiting instruction.

ii. Judge’s decision about whether or not to admit a piece of evidence is subject to “abuse of discretion” standard. Lots of leeway for judges.

III. Specialized Relevance Rules: Character Evidence —Character of A PARTY(404)

a. This may be relevant, but it’s generally barred. You can’t present evidence of a character trait to show that in the case at issue they acted in accordance with that character.

b. Three ways to show character evidence:

i. Reputation

ii. Witness’s opinion

iii. Prior bad acts

c. If a defendant offers evidence of his own character, prosecution can then offer evidence to rebut that. Opening the door.

1. Where character evidence is admissible, usually it’s just reputation and witness’s opinion that’s allowed. You can’t present evidence of specific acts. BUT, when you cross-examine a witness, you can ask about prior bad acts. You can’t prove it, so you’re stuck with their answer, but you can ask about it. The cross-examiner must have a good faith basis for the questions.

ii. [404(a)(2)(b)] Defendant can offer evidence of victim’s character trait (other than 412 sexual tendencies), and prosecution can

1. offer evidence to rebut that

2. offer evidence that defendant has the same trait

iii. Other relevant uses:

1. If bad acts are relevant for other reasons, you can PROVE them for that reason. (Not just offer them on cross-examination of character witness after defendant brings up his own character.) (405(b))

d. Defendant can offer evidence of victim’s character (minus rape shield stuff). But if he does, then the prosecution can offer evidence of victim’s character AND of defendant’s same character trait.

e. So: If defendant offers evidence of his own or of the victim’s character, the government may offer evidence of the defendant’s character.

i. For witness character evidence, see rules 607-609

IV. Habit evidence-Habit of A PARTY (406)

a. Permissible

b. Likely to be much less prejudicial; address course of conduct, not a moral judgment.

c. Habits are

i. Specific

ii. Regular

iii. Involuntary/less volitional

iv. Innocuous

V. Other types of relevant evidence that are not admissible (relating to what A PARTY does):

a. 407

i. Subsequent remedial measures (things you do to fix problems) cannot be used to prove negligence/guilt

b. 408

i. Settlement offers cannot be sued to prove liability/validity IF it’s a disputed claim. But if you own up to the fact that you did it and then discuss settlement, it’s not a disputed claim. Includes documents created for purpose of settlement.

c. 409

i. Offers to pay medical expenses cannot be used to prove liability

d. 410

i. Plea bargaining

e. 411

i. Liability insurance cannot be used to prove negligence

VI. Rape Shield Law (rape of A PARTY) (412)

a. An exception to otherwise admissible character evidence under [404(a)(2)(b)], which allows defendants to present evidence of the victim’s character, which the prosecution can then rebut or can show that the defendant has the same character

b. Can’t offer evidence of victim’s sexual behavior or sexual predisposition.

c. Exceptions to this (so, times when you CAN offer evidence of victim’s sexual conduct):

i. Criminal cases:

1. If offered to show that someone other than defendant was source of semen, injury, etc.

2. Other instances of sexual conduct with defendant if offered to prove consent

3. Evidence whose exclusion would violate defendant’s constitutional rights

ii. Civil cases

1. Only can offer evidence of sexual behavior/predisposition if it SUBSTANTIALLY OUTWEIGHS risk of harm

2. Or, can present evidence of victim’s reputation IF victim brought it up

d. Defendant’s similar crimes

i. If charged with sexual assault or child molestation, can introduce evidence of past crimes of sexual assault or child molestation. Addresses past crimes, not past convictions. (413-414)

ii. The standard for admitting is the conditional relevance standard-enough for a jury to reasonably find for the evidence. (Because it’s really conditionally relevant information-it’s only relevant if the defendant committed those prior acts.)

VII. Witnesses

a. Competency (of a witness)

i. Must have personal knowledge of the information AND

ii. Be able to take an oath and understand the obligation to tell the truth

b. Impeachment (of a witness OR defendant)

i. Ways to impeach

1. Character for truthfulness/untruthfulness (608) [every time a witness is in the witness box, character can be brought up]

a. Can use another witness to give reputation or opinion evidence; NOT evidence of specific bad acts

b. Can ask about prior bad acts only on CROSS-EXAMINATION; but cannot prove via extrinsic evidence. Must take their answer as is.

2. Prior criminal convictions (609)

a. Must be conviction for a crime that by its terms involves false statements (e.g. forgery)

b. OR a felony within 10 years (609(a)(1))

i. This is subject to 403 balancing by judge, though. (That prejudice does not outweigh probativeness.)

ii. If you’re trying to impeach the defendant himself, you must first show that the probative value outweighs the risk of prejudice.

Introduction

a. It is important to understand each rule’s legislative history, which may sometimes override an apparently plain and unmistakable meaning of the words in the rule. When a rule’s language and history both fail to make its meaning clear, look to the C/L for guidance

b. Why Study Evidence?

a. Every lawyer should know what is admissible in a case, especially litigators. Even a business lawyer should know what admissible evidence his client should keep on file and what evidence his client should destroy

b. There are differences in planning between states, which is important in determining which state to sue in

i. e.g. accident in NM and suit brought in VA. NM accident law will apply (i.e. negligence), and VA evidence law will apply (i.e. procedure)

ii. Different states have different privilege laws (e.g. some states have accountant-client privilege and some states don’t)

c. Sources of Evidence Laws:

a. Most laws come from the state legislature or the courts (when there’s a legislative enactment enabling the ct. to establish rules)

b. 5th Amendment –> person cannot be called to the stand in any case if the evidence might tend to incriminate that person

c. 6th Amendment –> the accused has the right, in a criminal prosecution, to be confronted w/ the witnesses against him

d. Original source of evidence was C/L and there are still areas where C/L applies:

i. In federal courts, the rules of privilege are decided by the C/L rules of privilege

d. Generally:

a. There is one set of rules throughout the j/d, but some only apply to criminal cases

b. Jury v. Nonjury cases – In jury cases, ct. has a lot of power to decide what to admit/not to admit, and COA is very deferential to that ruling. If evidence will be really prejudicial, ct. will keep it out. In nonjury cases, the judge never keeps out evidence b/c it’s prejudicial, b/c he’s the one who has to hear it to determine if it’s prejudicial. Judge can’t forget what he has heard.

c. We have rules based on what is admissible (lawyers cannot just put in w/e evidence they want) b/c:

i. That would allow for a filibuster — a lawyer would have people speak for hours until the case is thrown out. The jury would get restless and the other side will give up.

ii. Even if the evidence is relevant we should make sure we don’t waste time (e.g. by admitting evidence of something which has already been established)

iii. Some evidence can be really prejudicial, especially when unrelated (e.g. evidence D killed a man in a negligence action)

iv. Some evidence may be unreliable –> e.g. a witness testifying what his neighbor told him; we don’t allow hearsay evidence b/c then we cannot cross-examine that witness (6th Am.)

1. We also don’t allow infants or an elderly man with dementia to testify

v. Policy reasons –> we can keep out highly relevant and reliable evidence b/c we’d rather risk a bad result (not convicting a guilty person) than reveal the damaging info

1. e.g. want to protect Attorney -client privilege, spousal privilege; some states have a child-parent privilege

e. Tanner v. United States (Sup Ct. 1987) – p.7 –> FRE 606(b) (p.127)

a. Facts: Tanner argues the district ct. erred in refusing to admit juror testimony at a post-verdict hearing on juror intoxication during the trial. There is evidence they were drinking, doing cocaine, etc. and Tanner wanted a new trial.

b. Holding: Rule 606(b) doesn’t permit any inquiry into the internal deliberations of jurors (physical incompetence = internal)

i. However, “external”deliberations are allowed: e.g. (1) extraneous prejudicial info brought to jury’s attention (e.g. radio/newspaper account) or (2) some outside influence like a threat to juror’s life, (3) a mistaken verdict on the form; or (4) “chance verdicts”– some states allow evidence they flipped a coin

ii. Note: This rule only applies to jurors. A janitor can testify as to what he heard inside the deliberation room

c. Policy Rationale: (1) want to encourage full and drank discussions in jury room – jurors cannot function effectively if they’re later publicly scrutinized; (2) there should be a finality to litigation – lots of verdicts would not stand if internal deliberations were admitted; (3) jurors’ willingness to return an unpopular verdict and the community’s trust in a system that relies on the decisions of laypeople would be undermined by a barrage of post verdict scrutiny of juror conduct

d. Dissent: Drugs and alcohol are outside influences on jury members. Also, if the above-referenced policy considerations seriously threaten the constitutional right to trial by a fair and impartial jury, they must give way

Chapter 1: General Principles of Relevance

Probativeness and Materiality

a. FRE 401 (p.39) – “Relevant evidence”means evidence having any tendency to make the existence of any fact that is of consequence to the determination of the action more probable or less probable than it would be without the evidence.”

a. It is relevant even if it only helps a little bit (i.e. if it proves a sub-element of an element of the case, not the ultimate decision)

b. Probativeness (Logical Relevance): the evidence must have any “tendency to make the existence of that fact more probable or less probable than it would be without the evidence”—> (doesn’t need to prove anything conclusively, easy test)

a. Whether evidence is “probative”is determined by the judge and is usually not overturned. It depends on the judge’s background and he sees the world.

b. HYPO. Person in community committed hit-and-run. Police search around town and bring in one person. Only evidence they have is that when D opened the door and saw the cops, he took off. Probative of his guilt?

i. It depends on the situation. If it is a bad neighborhood and he’s running because of another crime he committed, then it’s not probative. Also, in a bad neighborhood, running from the cops may be the most sensible thing to do. If it’s a nice neighborhood, running from the cops has a whole different connotation.

c. Materiality: evidence is material if it bears on a “fact that is of consequence to the determination of the action”

a. e.g. whether a murder victim had a lucrative job is typically of no consequence to the D’s guilt or innocence. However, in a civil wrongful death suit arising out of the same attack, the victim’s lost earnings would be a material fact

i. Note: the victim’s salary would be probative of his earning potential, however.

d. HYPO. D charged w/ killing wife. Prosecution wants to put in evidence he had affair w/ secretary. Or assume prosecution wants to put in evidence his wife was having an affair w/ a co-worker. Are either relevant?

a. Yes, both are relevant b/c it shows he had a motive. But it cannot be used to convict him, b/c it doesn’t prove anything

b. But you can’t put in evidence that D did not have an affair b/c that’s too far removed from tending to show D’s innocent

b. FRE 402 (p.41) – All relevant evidence is admissible, except as otherwise provided by these rules. Irrelevant evidence is NOT admissible

c. Probativeness Problems

a. 1.1 (p.21) – D charged w/ murder of biz partner. When cops come to his house, they handcuff him and wife asks what’s going on. They explain, and she yells, “Murder? Where is the body? Show me the body.”Cops had yet to find the body. Relevant?

i. It is probative b/c the wife couldn’t have known the police didn’t have a body, so she must have known this via D, who hid body. This evidence is material too and makes the prosecution’s case (that he committed the murder) more probable.

b. 1.2 (p.22) – At D’s trial for robbery, a Gov. witness testified D took part in the crime. D calls a witness who testified the first witness told D’s witness he intended to implicate D falsely. Prosecutor then asks D’s witness if he and D were part of a secret prison organization which required members to lie and kill for e/o. Is this question relevant?

i. It is probative b/c if D’s witness is lying here (i.e. part of the secret organization), then he’s lying about how the Gov.’s witness was lying. If D’s witness is lying this makes it more probable that D committed the robbery

c. 1.3 (p.22) – D, charged w/ murder, said he was in bed at the time of the crime. He took a polygraph test. D wanted to admit as evidence that he was very willing to take the test. Is this relevant?

i. Courts don’t allow this evidence b/c D may know that the results are not admissible in court

ii. Note: info obtained from the lie detector tests themselves are never admissible; cts. don’t think they’re reliable. However, the police use the test in deciding whether to prosecute or not

d. Materiality Problems

a. 1.4 (p.23) – D charged w/ violation of statute saying it’s unlawful for anyone who was convicted of a crime punishable by imprisonment > 1 year to carry a gun interstate. D carried gun across WI-IL border and prosecutor showed evidence the D previously was convicted of assault/battery, and served 3 mo. (punishable up to 2.5 years). D testifies she didn’t know it was punishable up to 2.5 years. Is this testimony relevant?

i. No, testimony is irrelevant b/c whether or not she knew is not of consequence, according to the statute.

b. 1.5 (p.24) – D charged w/ intentional murder. He wants to put in the fact he couldn’t have formed the intent to kill b/c he was so intoxicated. However, the law in Montana says you cannot take intoxication into account. Evidence of high BAC relevant?

i. The evidence is irrelevant to show he acted “purposely” or “knowingly” in shooting the victim b/c of the statute. However, the evidence can be relevant to show someone else shot the victim b/c he was so drunk he didn’t have the capability to fire the trigger (kind of a longshot, but it gets this evidence admitted)

c. 1.6 (p.30) – A cop faced manslaughter charges, but claimed he shot victim out of self-defense. After a bloody gang slaying, he was searching for suspects near the scene and found the victim who was carrying a violin. He shouted to drop the case, but D says victim raised the case and “aimed”it at officer. Is the evidence of absence of a gun relevant?

i. Evidence is irrelevant b/c if D shot victim thinking he was dangerous, it doesn’t matter whether victim was innocent

e. United States v. James (9th Cir. 1999) – p.25

a. Facts: D charged w/ aiding and abetting a murder. She handed her gun to her daughter who shot the victim (D’s boyfriend). D claimed self-defense b/c of her knowledge that the victim was extremely violent and she was afraid for her life.

b. Evidence: D wanted to show he was violent and she had reason to fear him via testimony that he had actually done the violent acts that he bragged to her about.

c. Holding: Evidence was let in. Showing that he actually did all of these things makes it more likely he bragged about them (people tend to tell the truth), and that she is telling the truth, so the evidence is relevant.

i. If she was told these things, she would have reason to fear and act out of self-defense

ii. B/c the crux of her case rested on her credibility, and her credibility could be directly corroborated through this evidence, it was very relevant and would be unfair to keep out

Conditional Relevance

a. FRE 104(b) – Relevancy conditioned on fact. When the relevancy of evidence depends upon the fulfillment of a condition of fact, the court shall admit it upon, or subject to, the introduction of evidence sufficient to support a finding of the fulfillment of the condition

a. A piece of evidence can come in only if something else proceeds it, b/c it’s only valuable if something else is shown. The “something else”must be shown by a preponderance of the evidence (more likely than not)

i. Ct. needs to find that a reasonable jury could find more probable than not that the “something else”was shown

b. Only admissible evidence may be used to prove contested preliminary facts (unlike 104(a) which can use hearsay)

c. All evidence has conditional relevance, but most of the time it is obvious the condition is fulfilled

i. e.g. “D is the one who ran the red light”is conditional on the fact the person testifying was actually at the scene

d. 104(b) standard is stricter than a 401 standard: e.g. – Under a bare relevance standard, judge just asks whether outcome of the bond-reduction hearing had any tendency to make it more probable D had motive to kill. Under conditional relevance standard, evidence of the hearing only comes in if prosecutor introduced “sufficient evidence to support a finding”D heard about the hearing —> (but it’s not too much stricter of a standard)

e. HYPO. D accused of shooting victim, and at his arrest he has blood on his shirt. At time of trial, the shirt disappeared so they couldn’t test the DNA. D says it was his blood. Prosecutor wants to put in evidence that D refused to give it to cops.

i. For this evidence to come in, there needs to be evidence D knew about DNA/blood tests (which the ct. assumes)

b. Problem 1.7 (p.31) – Prosecutor wants to show the spouse’s wife was planning to tell the son that her husband was not the father. She had already told the father (not husband). Prosecutor says this shows D had motive to kill the wife (preventing her from telling the son).

a. However, this evidence is not relevant unless the husband knew she was going to tell the son

b. Some evidence that he knew she was going to tell the son is that she would likely not tell her son this critical info w/o first telling her husband, especially since she did tell the father. D calling the son “her son”to cops doesn’t prove anything b/c he probably knew it wasn’t his son, but didn’t know she was going to tell the son

c. Cox v. State (p.32)

a. Facts: D denied involvement in the shooting. D objected to testimony that 4 days prior, Cox’s close friend was not given a lesser sentence for molesting the victim’s daughter. D’s mom was at ct. and D hangs out at his friend’s house all the time.

b. Holding: The relevance of this testimony depends upon whether D knew what happened at the bond reduction hearing. If he knew, the info is relevant and extremely probative of the State’s theory D killed victim.

i. B/c D hangs out at friend’s house a lot and D’s mom was at the hearing, a reasonable jury could conclude D knew

ii. Courts are lenient about what gets let in because a jury can choose to disbelieve any fact presented

Probativeness Versus the Risk of Unfair Prejudice

a. FRE 403 (p.44) – Although relevant, evidence may be excluded if its probative value is substantially outweighed by the danger of: unfair prejudice, confusion of the issues, or misleading the jury, or by considerations of undue delay, waste of time, or needless presentation of cumulative evidence

a. Trial judge has a lot of discretion and is rarely overturned. Only FRE 609(a)(2) is exempt from FRE 403 (witness committing a past act or crime of dishonesty can come in automatically to impeach)

b. Strong presumption in favor of admissibility (harm must SUBSTANTIALLY outweigh the good)

c. Relevant evidence is inherently prejudicial, so only UNFAIR prejudicial evidence is excluded

b. Balancing test for keeping out prejudicial evidence: Weigh (1) waste of time, cumulative, distracting?; (2) alternative ways to present the evidence? (3) is it material to the issues of the case? (if it’s prejudicial to impeach a witness, that is usually less worrisome than something prejudicial to the main issue of innocence or guilt)

Photos and Other Inflammatory Evidence

a. State v. Bocharski (p.39)

a. Facts: Old woman killed in her home from 16 stab wounds to the head. Her neighbor is charged w/ murder. Prosecution wants to admit pictures of the body from the scene and autopsy (body had deteriorated before being discovered).

b. Holding: Ct. found the photos of the body were admissible, but photos of the skull were not. However, b/c the jury didn’t seem too affected by seeing the skull photos, the verdict is not overturned and there is no new trial.

i. Photos of a homicide victim’s body are generally admissible b/c the fact and cause of death are always relevant in a murder.

ii. Photos of the skill were not admissible b/c the manner of the victim’s death were not in issue and the photos failed to show the D’s missing knife caused the wounds. These photos were introduced primarily to inflame the jury.

b. Note that if a D doesn’t contest the fact that is of consequence, the exhibit’s probative value may be minimal and the gruesome photos have little purpose except to inflame the jury, so it would be prejudicial and ct. may keep it out b/c of D’s proffer

a. However, the general principle is a prosecutor can prove his case as he wishes, regardless of whether D admitted the issue

c. What can D do if he knows gory photos will be presented and he wants to keep them out?

a. Motion in limine to get them excluded (but it will be admissible if it’s relevant other than D’s admission); eliminate potential jurors who may be really sensitive; limit jury exposure by keeping photos outside jury room; ask for black and white photos; request before trial what prosecutor will admit, and forewarn the jury they will be seeing a lot of ugly photos

d. Class Video. Baby died and father prosecuted. Evidence was a picture of baby’s body found in shallow hole in ground. Prejudicial?

a. Ct. held yes, b/c officer who found baby can just testify where he found her. Prof says a lot of cts. would let it in, b/c it doesn’t show the baby being abused. Prof also dislikes calling the baby a “victim,”as the issue was whether it died of natural causes

e. Problem 1.8 (p.43)

a. Facts: D convicted of possessing an unregistered machine gun. Issue was whether the fun was “rapid-fire.”Both parties had conflicting evidence. D suggested Gov.’s test failed b/c of a malfunction or b/c internal parts were dirty or worn. In response, Gov. showed a photo revealing nothing of the gun’s interior. Photo had 9 guns in background, belonging to D’s roommate.

b. Even if the photos were relevant, this is too unfairly prejudicial, as it makes jury think D’s dangerous and owns the other guns

f. Class Video. Video simulation of murder. Should disparity in finances affect whether evidence can be let in?

a. Some judges say it’s unfair if D is unable to defend himself w/ a movie; other judges say money doesn’t make a difference

g. Commonwealth v. Serge (p.45)

a. Facts: D charged w/ killing wife. Prosecution is allowed to present its theory via a computer-generated animation (CGA).

b. Holding: The content of the CGA was neither inflammatory nor unfairly prejudicial. It did not include sounds, facial expressions or evocative or life-like movements. It was devoid of drama, preventing jury from improperly relying on emotions.

i. Monetary disparity between the parties is a relevant factor in the prejudicial analysis, as D does not have the finances to create a CGA of his own. However, the ct. concluded that the trial ct. safeguarded against potential prejudice by supplying a cautionary instruction before playing the CGA (e.g. “you are not to confuse art w/ reality”).

h. United States v. James, dissent (p.50)

a. The documents showing the victim said those things to D were unfairly prejudicial (i.e. the relevance of whether victim’s acts are true didn’t outweigh this portrayal of the victim as a bad man). They didn’t show anything about D’s state of mind b/c she didn’t see them. Jury will think they’re doing society a favor by getting rid of this bad man.

b. The trial judge decided it was inadmissible, and overruling this decision is so rare it shouldn’t be overturned in a close case.

i. Fuhrman Tapes (OJ Simpson case) – p.52

a. A big issue is the detective’s (Furman) hatred of blacks and interracial couples. He’s alleged to have framed OJ. Furman testified he hadn’t used the N word in 10 years, and the defense wants to show a tape of him using it repeatedly recently.

b. Ct. allowed only a few excerpts of Furman using the N word to play in front of the jury. The tape has probative value in proving he is an unreliable witness, but to show all 41 instances of him using the word is unfairly prejudicial (cumulative/distracting) b/c it detracts from the main issue at hand: whether OJ killed his wife

I. GENERAL

Appealing Evidentiary Rulings, FRE 103: FRE 103(a) provides that an evidentiary ruling will be grounds for reversal on appeal only for two reasons:
“Affects substantial right: The error must affect a “substantial right” of a party, meaning that the ruling made some difference in the outcome of the trial and

i. Timely objection: The party must have made a timely objection

ii. Offer of proof: Where the ruling excludes evidence, an “offer of proof” alerting the court to the substance of the evidence of the excluded evidence

Plain error an error so serious, and so obvious, that it can be grounds for reversal even though no objection w as made to it during trial.
Objections: an objection is the means by which a lawyer can interrupt the trial to oppose the introduction of evidence.
Purposes of objection:

i. If sustained, increased chances of winning trial by excluding harmful evidence

ii. If overruled, preserving argument for appeal

II. RELEVANCY

A. Is the Evidence Relevant?

Only Relevant Evidence Is Admissible, FRE 402: Only relevant evidence may be admitted. (First determine if it is relevant.
“Relevant Evidence” , FRE 401: Evidence is relevant if it has “any tendency to make the existence of a fact of consequence more probable or less probable than it would be without the evidence.”

i. Four Questions to Ask:

1. What is the evidence? (What is the EF/ evidentiary fact?)

2. To what proposition of fact is the evidence is directed?

3. Does that EF increase the probability of the proposition? (What is the IF/ inferred fact?)

4. Is the proposition a fact of consequence (FOC)? Does the proposition of fact connect with an essential element (EE)?

ii. “Fact of consequence: A proposition is a fact of consequence if it can be connected through inferential reasoning to one of the essential legal elements of the substantive law that governs the case.

B. Is the Probative Value Substantially Outweighed by Dangers?

Probative Value of Relevant Evidence v. Danger, FRE 403: “Although relevant, evidence may be excluded if its probative value is substantially outweighed by the danger of 1)unfair prejudice, 2)confusion of the issues, 3)misleading of the jury, or 4)considerations of undue delay, waste of time, or needless presentation of cumulative evidence.” 403 favors admissibility.
Probative Value: Probative value is the persuasive effect that the item of evidence will be likely to have on the jury’s thinking about the fact of consequence it is offered to prove.

i. Probative Value Factors:

1. strength of generalizations supporting each inference;

2. certitude of witness; AND

a. Courts do not count the witness’s credibility when estimating probative value. Evaluating the credibility of witnesses is for the jury, the judge’s role is to estimate the probative value of testimony if believed.

3. need for the evidence.

a. Need factors:

i. Centrality of the point to be proved

ii. The degree to which it is disputed by the opponent

iii. Alternative means of proving FOC

Dangers:

i. unfairly prejudicial;

1. emotional response;

2. bad person prejudice; AND

3. potential to be used in a way that violates a rule of evidence.

ii. confusion of the issues; evidence confuses the issues when it focuses the jury’s attention too closely on a factual issue that is not central to the outcome of the case.

iii. misleading the jury; Judge fears the jury will give the evidence more weight than it deserves or will draw a mistaken inference.

1. Examples: Video reenactment b/c jury may treat the documentation as the actual event. Scientific evidence can appear overpersuasive (i.e., lie detector test).

iv. undue delay, waste of time, and needless cumulative evidence.

Limiting Instructions, 105: When an item of evidence has a proper relevant use to prove a FOC but also creates the risk of an improper use (danger), the judge may give a limiting instruction that directs the jury to consider the evidence for its proper use. (But there is still the danger that the limiting instruction won’t be effective.)

III. LAYING THE FOUNDATION FOR PROOF

A. Standards of Proof

Preponderance of the Evidence Standard, 104(a): Under the 104(a) preponderance of evidence standard, the judge must actually be persuaded that the preliminary fact is more likely than not. In making its decision the court is not bound by the FRE except those regarding privilege. In general, all preliminary questions of fact are for the judge under FRE 104(a), unless they are necessary to the relevance of the offered evidence.
Prima Facie Standard, 104(b): Under the prima facie standard of 104(b), there are two steps: 1)first, there is a threshold decision by the judge whether a reasonable jury could conclude that the fact exists; and 2)second, if the judge admits the evidence it is left to the jury whether there is sufficient evidence that the preliminary fact is more probable than not.
Examples:

i. P is injured when his tire blows out; D claims that he warned P of the problem. The preliminary fact is whether P heard the warning; the conditionally relevant fact is the warning’s contents. The judge will decide whether a reasonable jury could find that P heard the warning; if he decides that the answer to this question is “yes,” he will let the jury hear the warning’s alleged contents, and it will be up to the jury to decide whether P really heard that warning and its contents.

ii. One witness testifies that he saw the perpetrator flee the crime scene wearing a red hat (EF1). The red hat is not relevant, unless another witness can testify that the red hat belongs to D (EF2).

104(b) Applies To:

i. Conditional Relevancy:

ii. FRE 602, Personal knowledge of matter:

iii. FRE 901, Identification or Authentication of Exhibit:

iv. FRE 1008 (we’ll read about in chapter 9)

v. FRE 404(b) second sentence, Specific acts for Noncharacter Use

vi. FRE 413-415, Specific acts of sexual misconduct to prove character and propensity

B. Preliminary Questions of Fact

Issues of Fact: If it is a preliminary question of law, the judge always decides. But if it is a preliminary question of fact, it depends on whether it is a relevancy based decision.
Non-Relevancy Based = Judge, 104(a): If an objection to admissibility is based on a technical exclusionary rule, any factual question needed to decide that objection belongs solely to the judge (i.e., hearsay, privileges, best evidence rule). Under FRE 104(a), when the judge makes such a finding he is not bound by the rules of evidence except those regarding privileges.
Relevancy Based: If the objection is that the evidence is irrelevant, the judge’s role may be more limited.

i. Ordinary Relevance Problem = Judge, 401: Ordinarily, a relevance objection may be decided without any finding of fact the judge merely has to decide under 401 whether, assuming the proffered fact is true, it makes some material fact more or less likely; this is purely a legal conclusion, so the judge handles it himself.

ii. Conditional Relevancy (“Conditional Admissibility” ) = 104(b): If fact B is relevant only if fact A exists, B is “conditionally relevant.” Under 104(b), first the judge decides whether reasonable jury could find that fact A exists, and if so, the jury will make the final decision whether fact A exists.

1. 104(b) vs. 401: 104(b) has a higher standard of proof than 401. Under 401, the standard is whether the line of reasoning has “any tendency” to affect the probabilities of a fact of consequence. However, 104(b) requires the judge to apply the higher threshold of “evidence sufficient to support a finding.”

2. Judge May Allow Early: The judge may allow the conditionally relevant fact into evidence prior to his showing of the preliminary fact; the conditionally relevant evidence is said to be admitted “subject to connecting up.”

3. Judge May Instruct Jury: After the judge decides it meets the sufficiency standard, he may instruct the jury that it must decide the preliminary fact question before it may considers the offered item of evidence to which it pertains.

C. Laying the Foundation for Witnesses, FRE 601-603:

Requirements:
competency, FRE 601;

i. The court will look at the witness’s ability to understand and answer questions. (i.e., child could have difficulty with these.)

first-hand, personal knowledge (104(b) standard), FRE 602; AND

i. Prima facie standard: 602, prima facie standard evidence sufficient to support a finding. Judge must find that a reasonable jury could find that it’s more probable than not that witness has first hand knowledge.

ii. Third-party hearsay: a party does not have personal knowledge if he was merely told of something. (Example: you have a broken arm. D has personal knowledge of his arm hurting, but unless he sees the x-ray and it is explained to him, he doesn’t have personal knowledge that he has a broken arm.)

ability to take oath, FRE 603:

i. The witness must be able to promise to testify truthfully. Must be competent to understand consequences of lying under oath. (i.e., child might have difficulty with this.)

D. Laying the Foundation for Exhibits, FRE 901

1. Authentication or Identification, 901 = Prima Facie 104(b) standard: “The requirement of authentication or identification as a condition precedent to admissibility is satisfied by evidence sufficient to support a finding that the matter in question is what its proponent claims.” 901(a). Thus, authentication and identification is decided under a 104(b) prima facie standard.

a. Two Step Process:

i. Threshold Decision by Judge: So long as the judge determines that a jury could reasonably find in the proponent’s favor, the judge will find that FRE 901(a) has been satisfied.

ii. Final Decision by Jury:

2. “Real” v. “Demonstrative Evidence”

a. “Real: Real evidence is a tangible exhibit that played some actual role in the events that are in dispute.

i. Example: a knife used in a fatal stabbing.

b. “Demonstrative: demonstrative evidence refers to exhibits that reproduce or depict persons, objects or scenes that are connected to the litigated events in the case.

i. Examples are photographs, models, diagrams, or drawings. (Thus, it’s different than real evidence: real evidence knife; demonstrative evidence photograph of knife)

c. Significance of distinction: The foundation requirements needed to authenticate the two types of evidence are different.

3. Methods of Authentication

a. Real Evidence: for real evidence, authentication is generally done in one of two ways:

i. readily or uniquely identifiable; OR

1. Example: I found the knife at the stabbing, and marked it with my initials; the knife you have just shown me has my carved initials, so it must be the knife at the murder scene.

ii. chain of custody: If there is no readily or uniquely identifiable characteristic, the item’s chain of custody must be demonstrated. This means that every person who handled or possessed the object since it was first recognized as being relevant must explain what he did with it.

1. Complete Chain of Custody is Not Always Required, 901(a): the complete chain of custody is not always required to to identify real evidence or to show its unchanged condition. The question is whether jury could reasonably find that exhibit in question is what it is claimed to be.

a. Example: Police seize meth from D’s motel room. Evidence custodian couldn’t recall who delivered the drug for analysis, but it was allowed. Concern went to the weight of the evidence, not its admissibility.

iii. Objections Unfairly Prejudicial or Misleading, FRE 403: claims that real evidence is unfairly prejudicial or misleading are heard under 403.

b. Demonstrative Evidence: Two steps:

i. Have witness identify demonstrative evidence through testimony; AND

ii. have witness state that it is a fair and accurate depiction of real object (authentication).

1. Example: The prosecution offers a photograph or drawing of the knife that was allegedly found at the crime scene. A witness familiar with the real knife might testify, “I recognize the knife in the photo b/c of the initials KRS on it. It is the knife I found at the crime scene. It is a fair and accurate portrayal of the knife.”

iii. Objections Misleading, Prejudicial, FRE 403: Claims that a demonstrative exhibit is misleading or prejudicial (i.e., false impression) would be decided under 403.

4. Other Types of Evidence and Methods of Authentication, 901(b)

a. Determine Relevancy First: Always remember, the starting point of your analysis of the requisite foundation should always be relevancy.

b. Written Documents: Written documents are identified by testimony that identifies the author or source of the document, typically by using the signature, the contents, the location of the document, or other circumstances.

i. Identification by Signature, 901(b)(1)-(3): observation of the creation of a document (901(b)(1), identification based on familiarity with handwriting 901(b)(2), or comparison of handwriting 901(b)(3). (A signature alone is not enough the proponent must make some independent showing that the signature was made by the person who the proponent claims made it.)

ii. Distinctive characteristics, 901(b)(4): A writing’s distinctive characteristics, or the circumstances surrounding it, may suffice for authentication.

iii. Authentication of Ancient Documents, 901(b)(8): If a writing 1)is more than 20 years old, 2)is in a place where it would likely be if it were authentic, and 3)the document is in such condition as to create no suspicion concerning its authenticity, it may be authenticated pursuant to FRE 901(b)(8) as an ancient document.

1. Example: German documents generated during World War II (more than 20 years old) that implicate current US citizens in Nazi war crimes have been authenticated by their storage location (Berlin) and by the expert testimony of historians who specialize in Nazi-era German history, policies, and practices.

Electronic Documents: Writings created by new technologies are identified and authenticated by analogies to Rule 901(b) illustrations (i.e., the “Written Documents” section of outline). Emails can be identified by the electronic address that they bear and by the electronic signature itself.

Cases on Evidence

Intro to Evidence Law

§ Trial Procedure

à˜ Pretrial Evidentiary Motions: motions in limine

o Asking to exclude evidence before trial based on what anticipate evidence to be.

à˜ Jury selection -> voir dire -> preemptory challenges for cause

o Opening Statements à Roadmap no argument/ P first b/c have burden of proof

à˜ Gov’t or P’s case-in-chief

o The party, not the ct, controls order of evidence & presentation of witnesses.

o Direct exam non-leading questions only

o Cross-examination leading permitted

o Re-direct

o Attorneys object to evidence, not the court. Objections can be waived not a basis for error if waived.

à˜ D moves for directed verdict

à˜ D’s case-in-chief

à˜ Closing argument Gov/P first, then D, then Gov/P again.

à˜ Jury Instructions

o Instructions of LAW that are applicable to the case. Largely created by attorneys and/or taken from model jury instruction books. Each side submits their instructions and each Attorney must object to anything in the instructions they feel state the law incorrectly.

§ Truth v. Justice v. Efficiency

§ Rule 103: Effect of Erroneous Rulings

à˜ Most trial judge decisions on evidence are reviewed for abuse of discretion

o Won’t be reversed unless extremely grossly in error

o Even if CoA finds abuse of discretion, they can find that the erroneous ruling was harmless

à˜ A substantial right of a party must be affected à could the error have affected the outcome of the case

à˜ You also must timely object to evidence; you must make a contemporaneous objection, objection it now so it can be fixed

o Exception: plain error à “particularly egregious,” a “miscarriage of justice” resulted from the error, or the error deprived the D of a fair trial

§ Chances of reversal on the ground of evidentiary error are very slim
A. Role of the Trial Judge
i. The Trial Judge’s Authority
ii. The Trial Judge’s Discretion
o U.S. v. Walton

§ Great deference to trial judge’s determination of admissibility of evidence
o Bandera v. City of Quincy

§ Bandera sexually assaulted; one of her witnesses was also sexually assaulted & testified about how Bandera felt; City appeals the admission of the opinion testimony

§ Relevant Evidence: Rule 401

à˜ Test: Evidence is relevant if: (a) it has any tendency to make a fact more or less probable than it would be w/o the evidence; AND (b) the fact is of consequence in determining the action. (“Just has to be a brick in the wall, not the whole wall” )

§ Irrelevant evidence is never admissible

§ Relevant evidence is generally admissible à lots of exceptions

à˜ Admissible unless any of the following provides otherwise:

o US Constitution

o A federal statute;

o These rules; or

o Other rules prescribed by the US Sup Ct

à˜ Biggest exception: Rule 403

o Unfair prejudice

o Confusion of the issues

o Misleading the jury

o Undue delay

o Waste of time

o Cumulative

§ Ct overrules motion in limine/ Renews objection in trial and overruled

§ This was plain error but unlikely to have affected the outcome though. Harmless error b/c what witness said was mild in comparison to what the P actually said about her experience.

§ Side issue: App Ct says we don’t know why he objected, it wasn’t specific enough

à˜ A motion in limine was made about this, but was deferred to be discussed later & when the testimony was later offered, the lawyer did not object again & therefore failed to preserve the objection so the review was at most for plain error.

à˜ 103(a)(1) timely objection stating the specific ground of objection, if the specific ground was not apparent from the context

à˜ If any doubt when judge overrules objection say, “for the record the basis for my objection was…”

· Contemporaneous objection rule??

Chapter 2 Relevance
B. Relevance and Irrelevance

§ TRIAL JUDGE DECIDES Q’s OF RELEVANCE

§ Low threshold for relevance!

§ Always the starting point:

à˜ Irrelevant evidence is NEVER admissible and relevant evidence generally is

o The first has no exceptions; the second has many.

§ Relevant evidence: helps persuade the trier of fact of the existence or nonexistence of some fact that is germane to the dispute b/w the parties

§ All evidence tends to prove or disprove some fact, but in a judicial trial the substantive law, or perhaps the state of the pleadings, circumscribes those facts that have legal consequences

§ The relevance or irrelevance of a piece of evidence is determined by logic and experience, not by legal rules. Evidence is relevant if it is rationally probative in any way.

§ FRE 401: Relevant evidence means evidence having any tendency to make the existence of any fact that is of consequence to the determination of the action more probable or less probable than it would be w/o the evidence

§ Evidence is relevant only if

à˜ 1. it tends to prove/disprove a proposition of fact that

à˜ 2. is of consequence under the substantive law as made applicable by the pleadings

§ Evidence is relevant only if it supports a factual proposition that is “of consequence” to the outcome at trial

§ “a brick is not a wall” ; the Q at the conclusion of the trial is whether all items of evidence taken together meet the applicable std

§ Test of ordinary relevance:

à˜ Whether the evidence offered has any tendency to make a consequential fact more or less
o Knapp v. State

§ 1st degree murder case

§ D offered a self-defense theory in killing a marshal. He had heard that the marshal had killed a man when arresting him & was afraid he would be killed as well.

§ Evidence in dispute: state offered evidence about the falsity of the previous story. The appellant contended allowing this was error the issue was whether he had actually heard the story, not to its truth.

§ Evidence allowed b/c it tended to discredit the appellant. Shows his story is less probable especially since he couldn’t point to his informant of the info.
o US v. Dominguez

§ FACTS

à˜ Dominguez was a US Customs Officer. Found guilty of kidnapping, robbing, & murdering Mitri when he tried to carry 700K into the US

à˜ Evidence at issue: D says evidence that he owned a gun was irrelevant and prejudicial b/c he had to own one for his profession (customs officer)

o Gov’t showed that gunshot killed Mitri & then introduced evidence showing (1) D owned a gun; (a) Week after M’s death, D asked a friend to bring his gun to a gun shop to have barrel replaced; (3) shop-owner saw scratches on barrel, which could have been left by an attempt to remove the barrel; and (4) the shop-owner repaired the barrel, but didn’t replace it.

§ Relevant evidence: having any tendency to make the existence of any fact that is of consequence more probable or less probable than it would be w/o the evidence (401)

§ CT: replacement effort makes guilt more probable à evidence relevant

à˜ The logical connection of above shows guilt is more probable than had there been no replacement effort so it is relevant.

§ Std of probability: more probable than it would be w/o the evidence

§ An item of evidence need not prove conclusively the proposition for which it is offered.

§ Affirmed
o State v. Larson

§ Larson riding horse w/ 5yo Perry when horse reared & fell backward & crushed Perry, killing her

§ Owner of horse warned D the horse was inexperienced and hot-blooded

§ 3hrs after accident D’s BAC was .17 and a forensic scientist established that his BAC was b/w .20 and .27 at the time of the accident

§ Compared D’s BAC w/ what has been determined to impair a person’s ability to drive an automobile and prosecutor said D’s BAC was 3x the level that impairs one’s ability to drive an automobile (state did this to give the jury something to compare the level to)

§ Evidence at issue: D contends that the level of blood alcohol that impairs a person’s ability to drive is irrelevant to his conduct relative to a high-spirited young horse

§ CT: evidence showed the alcohol impaired his jdmt and reactions so BAC was relevant

à˜ Comparison is also relevant bc it aided jury in evaluating his level of intoxication

à˜ Probative value outweighs any prejudice to the D

§ Affirmed
C. Probative Value and Prejudice

§ FRE 403. Relevant evidence is inadmissible if the trial judge determines that the probative value of the evidence is substantially outweighed by considerations of unfair prejudice, confusion of the issues, misleading the jury, undue consumption of time, or unfair surprise

§ Std of review: clear abuse of discretion

§ Vast bulk of evidence law consists of exceptions to the general proposition that relevant evidence is admissible

à˜ Most important exception: giving trial judges broad discretion to exclude evidence that is more trouble than it is worth

o Will impair rather than assist the jury’s search for the truth OR

o It will take up too much time

§ Reasons for excluding evidence:

à˜ Risk of unfair prejudice: undue tendency to suggest decision on an improper basis (commonly an emotional basis)

o Look at effectiveness of a limiting instruction on whether to exclude on grounds of unfair prejudice.

à˜ Confusion of the issues

à˜ Misleading the jury

à˜ Wasting time

§ What about unfair surprise? Usu granting a continuance is more appropriate remedy than exclusion of the evidence

§ Balance the probative value of & need for the evidence against the harm likely to result from its admission

§ If the balance is close à federal rules favor admissibility

à˜ 403 says SUBSTANTIALLY outweighs probative value
o US v. Noriega (p. 26)

§ FACTS

à˜ D served as commander of Defense Forces in Panama; Indicted on drug related charges saying D used his position to help Columbian drug traffickers smuggle cocaine thru Panama and into the US

à˜ Before trial, D gave notice of his intent to use classified info regarding intelligence work for the US to rebut the gov’ts assertion that he had unexplained wealth

o US said it was 320K and D said it was really about 10M and he should be allowed to disclose the tasks he performed for the US

o Dist. Ct said that disclosing the info about the tasks performed was irrelevant and the tendency that such evidence would confuse the issues before the jury substantially out-weighed any probative value it might have had

o D was allowed to present evidence of the fact, amts, time, source, and method of conveyance of $$ he alleged he received from the US but he didn’t even submit the evidence regarding the money he allegedly received from the US bc he said it wouldn’t have appeared credible to the jury absent details regarding the actual services he performed

§ ISSUE

à˜ Did the probative value of the excluded evidence (info regarding purposes for which the US paid D) outweigh the harm likely to result from its admission à specifically confusion of the issues (403)

§ CT: Info regarding the purposes for which the US paid D potentially had some probative value (but was relatively marginal)

à˜ Would have helped the jury determine which of the 2 pymts was more credible

à˜ It was relevant!

à˜ BUT, Admission of evidence would have shifted unduly the focus of the trial from allegations of drug trafficking to matters of geo-political intrigue

o i.e., It would confuse the jury as to the crime

à˜ Dist. Ct did NOT abuse its discretion when it determined that the probative value of the proffered material was outweighed substantially by the confusion of issues its admission would have created

Hearsay

Rational & Meaning
Rule 801. Definitions
The following definitions apply under this article:
(a) Statement.
A “statement”is (1) an oral or written assertion or (2) nonverbal conduct of a person, if it is intended by the person as an assertion.
(b) Declarant.
A “declarant”is a person who makes a statement.
(c) Hearsay.
“Hearsay” is a statement, other than one made by the declarant while testifying at the trial or hearing, offered in evidence to prove the truth of the matter asserted.
(d) Statements which are not hearsay.
A statement is not hearsay if–
(1) Prior statement by witness. The declarant testifies at the trial or hearing and is subject to cross-examination concerning the statement, and the statement is (A) inconsistent with the declarant’s testimony, and was given under oath subject to the penalty of perjury at a trial, hearing, or other proceeding, or in a deposition, or (B) consistent with the declarant’s testimony and is offered to rebut an express or implied charge against the declarant of recent fabrication or improper influence or motive, or (C) one of identification of a person made after perceiving the person; or
(2)Admission by party-opponent. The statement is offered against a party and is
(A) the party’s own statement, in either an individual or a representative capacity or
(B) a statement of which the party has manifested an adoption or belief in its truth, or
(C) a statement by a person authorized by the party to make a statement concerning the subject, or
(D) a statement by the party’s agent or servant concerning a matter within the scope of the agency or employment, made during the existence of the relationship, or
(E) a statement by a coconspirator of a party during the course and in furtherance of the conspiracy.
The contents of the statement shall be considered but are not alone sufficient to establish the declarant’s authority under subdivision (C), the agency or employment relationship and scope thereof under subdivision (D), or the existence of the conspiracy and the participation therein of the declarant and the party against whom the statement is offered under subdivision (E).

Rule 802. Hearsay Rule
Hearsay is not admissible except as provided by these rules or by other rules prescribed by the Supreme Court pursuant to statutory authority or by Act of Congress.

Hearsay Exceptions
There are 2 broad categories of statements transformed into nonhearsay by Rule 801(d)
1. Certain prior statements by a testifying witness
2. Statements made by a party or a party’s representative (“Party Admissions” ) offered in evidence by that party’s opponent.

Prior Statements of a Testifying Witness
1. Specified Prior Inconsistent Statements
2. Specified Prior Consistent Statements
3. Prior Identification of a Person

Note, however, that each of the three exempted classes of prior statements by a testifying witness has a basic requirement in common: the prior statement is not converted from hearsay to nonhearsay unless the “declarant testifies at the trial or hearing and is subject to cross-examination concerning the prior statement.

Prior Inconsistent Statements
A prior inconsistent statement used only to impeach does not violate the hearsay rule because it is not offered for the truth of the assertion it contains. Rather, the earlier inconsistency is offered to show that at another time the witness has given a conflicting version of the same event and therefore his account from the witness stand is not reliable.

Grand jury hearings or proceedings fall squarely within Rule 801(d)(1)(A)

Rule 801(d)(1)(A)’s requirement of an oath and exposure to a perjury charge. First, these 2 requirements add to the likelihood that the declarant will tell the truth. Second, statements given under oath and subject to a perjury penalty are almost invariably recorded by a Court reporter or some other official acting on the government’s behalf.

Police interrogations do not qualify as proceedings under Rule 801(d)(1)(A). The language of the rule, which embraces prior inconsistent statements made at a trial, hearing, or other proceeding, or in a deposition, implies a formal inquiry, officially sanctioned by statute, regulation, or court rule, and conducted by a person or body that routinely receives witnesses’ testimony under oath subject to the penalty of perjury.

Prior Consistent Statements
There is 1 circumstance in which a witness’s prior consistent statement has enhanced probative value. When a cross-examiner directly or by fair implication charges that witness’s testimony is a “recent fabrication” or is the product of an “improper influence or motive,” a prior consistent statement may have considerable probative force to rebut this charge. It is imperative, however, that the prior consistent statement predate the alleged corrupting influence or purported fabrication.

There is no requirement that the prior consistent statement be made “under oath subject to the penalty of perjury”which is a requirement for a prior inconsistent statement proffered under Rule 801(d)(1)(A).

Factual determinations of the trial judge are important under all 3 subsections of Rule 801(d)(1)(B) prior consistent statements. That is because it is often unclear exactly when the influence, motive, or incentive to falsify arose.

Prior Identifications
An earlier out-of-court identification is more reliable than one made in the courtroom. The earlier identification is closer in time to the event that is the subject of the litigation than is a similar identification made at the trial

More about Evidence

Witnesses Qualification/Selection

Compulsory Process

English Law: the King could summon witnesses (and hold them until the trial); the defense had no such right… later the Treason Trials Act presented reform (William Ireland’s trial)

US Constitution: Sixth Amendment the accused shall enjoy the right to compulsory process in calling witnesses….

Point of Compulsory Process Clause –

1. Cross Examination, in addition to preventing bad witnesses/information (perjury), increases the accuracy of the information presented

2. Better opportunity for a fair trial, jury to reach the right decision

3. Better opportunity to present a full and robust defense

Chambers v. Mississippi: illustrates theory behind FRE 607 (Any party, including one who calls a witness , may attack the witness’s credibility)

Rule 606: Justifications

· Limit harassment of jurors

· Finality

· Legitimacy

Alternative Solutions

· Voir dire

· Observable by other parties

· Non-testimonial evidence

Rule 606. Competency of Juror as Witness

(a) At the trial. A member of the jury may not testify as a witness before that jury in the trial of the case in which the juror is sitting. If the juror is called so to testify, the opposing party shall be afforded an opportunity to object out of the presence of the jury.

(b) Inquiry into validity of verdict or indictment. Upon an inquiry into the validity of a verdict or indictment, a juror may not testify as to any matter or statement occurring during the course of the jury’s deliberations or to the effect of anything upon that or any other juror’s mind or emotions as influencing the juror to assent to or dissent from the verdict or indictment or concerning the juror’s mental processes in connection therewith. But a juror may testify about (1) whether extraneous prejudicial information was improperly brought to the jury’s attention, (2) whether any outside influence was improperly brought to bear upon any juror, or (3) whether there was a mistake in entering the verdict onto the verdict form. A juror’s affidavit or evidence of any statement by the juror may not be received on a matter about which the juror would be precluded from testifying.

ANALYSIS OF EVIDENCE FOR ADMISSIBILITY

Relevance

0. Probative

· Does it make some fact more or less probable?

1. Material

· Is it a fact of consequence to the determination of the action?

Conditional Relevance

· Is there sufficient admissible evidence such that a judge would rule that a reasonable jury could find by a preponderance of the evidence?

· Huddleston v. United States (Huddleston standard) – evidence should be admitted if there is sufficient evidence to support a finding by the jury that the defendant committed the similar act

v By a preponderance of the evidence

v Judge makes the decision

v Only admissible evidence allowed

If relevant, do any exceptions bar it (FRE 407-411)?

v Subsequent remedial measures

· To prove culpability, product liability, warning defects or strict liability?

ü Does not apply to third party’s subsequent remedial measures

· Used to controvert feasibility? Has the feasibility been controverted?

ü Consider both narrow and broad definition of feasibility

v Compromise offers, actual compromise or statements made during negotiations used to show liability, invalidity of or amount of claim that is disputed to validity or amount?

· Exception: statements made in negotiations with state agency See details

· Exception: when rule is being used as a sword rather than a shield

v Compromise offers, actual compromise or statements made during negotiations being used to impeach through a prior inconsistent statement or contradiction?

· Exception: statements made in negotiations with state agency

v Offering or furnishing payment for medical expenses being used to prove liability?

v Liability insurance being used to show culpability?

v Statement made during the course of plea discussions offered for any purpose other than fairness of context or in a criminal proceeding for perjury?

Prejudice

1. Is the probative value substantially outweighed by danger of unfair prejudice, misleading or confusion of the jury or waste of time?

v Monetary disparity may be a factor in determining unfair prejudice

o Ex: Commonwealth v. Serge, CGA

Unfair Prejudice: An undue tendency to move the tribunal to decide on an improper basis, commonly an emotional one

· Test: Does it prevent defendant from putting forth a reasonable defense? (Case by case analysis is required)

Methods to Reduce Unfair Prejudice

· Jury instructions

· Stipulations

· Size and crop presented to jury

· Duration or number of times presented to jury

· Voir dire

· Use of other similarly relevant evidence

Situations w/Potential for Unfair Prejudice

1. Evidence of Flight

v Admission by conduct through inference

o From defendant’s behavior to flight

o From flight to consciousness of guilt

o From consciousness of guilt to consciousness of guilt concerning the crime charged

o From consciousness of guilt concerning the crime charged to actual guilt

v May be difficulty with inference if multiple crimes had been committed

o Inference from consciousness of guilt to consciousness of guilt concerning crime charged

o May be fleeing due to unrelated crime

o Requires defendant to put in evidence of other crime

o Inadmissible due to unfair prejudice

v More remote in time crime was, greater chance that flight is less probative of particular crime

v Evidence of flight is only marginally probative

o Person may leave scene of crime or jurisdiction for multiple reasons

v Alternatively one may submit evidence of non-flight

2. Statistical Probability

v Must have foundation and statistical independence

o May not rest on false data or mathematical principles

o Jurors must have ability to spot flaws

o Admissible evidence – DNA, fingerprints, bite marks, weapons

v Highly prejudicial and low probative value

v Jury’s will give disproportionate weight to probability evidence

o Even if probability was accurate one could not conclude that defendant was guilty

§ EX: convicting X because a witness saw either X or X’s twin

Stipulations

Stipulations are a compromise to battle prejudice
Disadvantages

1. Stipulated fact could slip

2. Jury distrust and resultant imagination

3. May bring more attention to omitted details

A party may not stipulate or admit his way out of the full evidentiary force of case as the other party chooses to present it

· To permit party to present to the jury a full story

· To satisfy expectations about what proper proof should be

· Status may be an exception to this rule

· Stipulations as to a status of a convict leaves little gap in narrative due to remote nature to crime charge

Rule 401. Definition of “Relevant Evidence”

“Relevant evidence” means evidence having any tendency to make the existence of any fact that is of consequence to the determination of the action more probable or less probable than it would be without the evidence.

Rule 402. Relevant Evidence Generally Admissible; Irrelevant Evidence Inadmissible

All relevant evidence is admissible, except as otherwise provided by the Constitution of the United States, by Act of Congress, by these rules, or by other rules prescribed by the Supreme Court pursuant to statutory authority. Evidence which is not relevant is not admissible.

Rule 403. Exclusion of Relevant Evidence on Grounds of Prejudice, Confusion, or Waste of Time

Although relevant, evidence may be excluded if its probative value is substantially outweighed by the danger of unfair prejudice, confusion of the issues, or misleading the jury, or by considerations of undue delay, waste of time, or needless presentation of cumulative evidence.

5 Factors set forth to determine if evidence should be admitted when analyzing under 403

– Nature of the crime (probative value)

– Time of conviction and subsequent history (probative value)

– Similarity between prior crime and the charged crime (high danger of unfair prejudice due to jurors’ likely use for an impermissible purpose)

– Importance of defendant’s testimony (don’t want to chill the defendant’s right to testify, can cause unfair prejudice)

– The centrality of the credibility issue

Rule 104. Preliminary Questions

(a) Questions of admissibility generally. Preliminary questions concerning the qualification of a person to be a witness, the existence of a privilege, or the admissibility of evidence shall be determined by the court, subject to the provisions of subdivision (b). In making its determination it is not bound by the rules of evidence except those with respect to privileges.

(b) Relevancy Conditioned on Fact. When the relevancy of evidence depends upon the fulfillment of a condition of fact, the court shall admit it upon, or subject to, the introduction of evidence sufficient to support a finding of the fulfillment of the condition.

CHARACTER EVIDENCE

Basics

· Character evidence of a pertinent trait of the accused or victim is initially admissible only by accused

v Subject to limitations of Rule 412 Rape Shield

v Only applicable to criminal cases

v Once introduced, the door is opened for the prosecution to:

ü Offer evidence of the same trait of accused to rebut the character evidence introduced

ü Offer evidence of the same trait of the victim to rebut character evidence introduced

ü Offer evidence of a trait of the accused if the accused has offered evidence of that same trait of the victim, even if the accused did not put his character at issue

v If accused claims self-defense in a homicide, evidence of the trait of peacefulness of the victim is admissible to rebut evidence that victim was the first aggressor

ü Admissible even if the accused offers non-character evidence

Rationales:

Jury may accord the character evidence excessive weight
Jury may convict irrespective of guilt for present charge, because a bad person who deserves it
Mini-trial for largely unrelated evidence

History:

People v. Zackowitz: Evidence intended to show a person is “dangerous and vicious” (or has another certain character trait) may not be admitted if it is intended to establish that his character trait makes him more likely to have committed a crime.

– No Rule 404 at time of the decision, Cardozo’s opinion states that the evidence must be excluded because a person’s propensity to commit the type of crime charged cannot be entered since it is not “evidence of the events that actually transpired” .

o Consider: Same evidence could possibly have been introduced as to his mental state in specifically choosing the particular weapon he did; or could have been introduced (via his statement to the police)to impeach his statement at trial

Rule 404. Character Evidence Not Admissible To Prove Conduct; Exceptions; Other Crimes

(Subject to 403 determination whether undue prejudice outweighs probative value in view of other means of proof and factors appropriate or making decisions of this kind Advisory Committee Note)

(a) Character evidence generally.–Evidence of a person’s character or a trait of character is not admissible for the purpose of proving action in conformity therewith on a particular occasion, except:

(1) Character of accused.–In a criminal case, evidence of a pertinent trait of character offered by an accused, or by the prosecution to rebut the same, or if evidence of a trait of character of the alleged victim of the crime is offered by an accused and admitted under Rule 404(a)(2), evidence of the same trait of character of the accused offered by the prosecution;

(2) Character of alleged victim.–In a criminal case, and subject to the limitations imposed by Rule 412, evidence of a pertinent trait of character of the alleged victim of the crime offered by an accused, or by the prosecution to rebut the same, or evidence of a character trait of peacefulness of the alleged victim offered by the prosecution in a homicide case to rebut evidence that the alleged victim was the first aggressor;

(3) Character of witness.–Evidence of the character of a witness, as provided in Rules 607, 608, and 609. (Impeachmeant)

(b) Other Crimes, Wrongs, or Acts.–Evidence of other crimes, wrongs, or acts is not admissible to prove the character of a person in order to show action in conformity therewith. It may, however, be admissible for other purposes, such as proof of motive, opportunity, intent, preparation, plan, knowledge, identity, or absence of mistake or accident, provided that upon request by the accused, the prosecution in a criminal case shall provide reasonable notice in advance of trial, or during trial if the court excuses pretrial notice on good cause shown, of the general nature of any such evidence it intends to introduce at trial.

Interpretation of 404(b)

· Proof of knowledge can encompass specialized knowledge or notice

· “Modus operandi”

v Similarities between the two crimes must be so distinctive to produce inference that nobody else could have committed this crime

ü Unique criminal fingerprint

v Totality of comparison analysis; need not be exact replica

Reverse 404(b) Evidence

Using evidence of other crimes to exonerate defendants, instead of implicating them
Lower standard of similarity needed for this due to no danger of unfair prejudice
Must have a fact pattern conducive to such a finding

v Ex: custody, strong alibi, mistaken identification

Not allowed to use evidence to implicate third party

v “It’s not me”v. “It’s someone else”

v Reverse 404 does not use general character evidence

Evidence of Prior Acts Allowed (Getting Around the Propensity Box):

Constitute a part of the transaction that serves as the basis for the criminal charge
Necessary to permit the prosecutor to offer a coherent and comprehensible story regarding commission of crime; despite its danger as being perceived as character evidence

v Res gestae or “narrative integrity”

ü Ie. russian roulette – Allowed to describe gun but not why was memorable

· Would be unfairly favorable to the defendant if all entered

More cases on Evidence

o Relevance Generally

Evidence Fact Issue in Dispute
Probative means that the evidence makes it more or less probable that a fact exists.
That fact must be relevant, of consequence to the issue in dispute.
Standard of evidence for the federal rules is very lenient, almost anything is relevant.
Problem 1.1: Show me the body! Murderers wife shouts show me the body when arrested. This is relevant because the police had not found the body yet and indicated, this indicated she had some knowledge of the murder, enough to be aware the police were unlikely to be able to find the body.
Problem 1.2: Guy on trial for robbery, government witness called who testifies he was a robber. Defense puts up witness saying government witness told him in prison he was going to lie and say defendant did it. On cross prosecution asks defense witness were he and defendant part of prison gang with creed to lie and kill for each other. Relevance is that defense witness is more likely to be lying on behalf of defendant, and furthermore if he is lying its more likely defendant was robber.
Problem 1.5: Guy murders two people while drunk. Got drunk willingly, wants to argue he didn’t knowing kill the other two. Montana law does not allow to present evidence of intoxication to mitigate intent. It is relevant though, especially to premeditated murder aspect, but not unconstitutional to bar it.

United States v. James (9th Cir. 1999)
James met boyfriend Ogden. He told her stories about how he messed people up, including killing one guy and stabbing another with a pen.
James saw Ogden get violent. Her daughter fought back against him and he never fought back, even though she was only fourteen.
Daughter killed Ogden, with gun her mom James handed to her.
Both James and daughter believed Ogden’s stories but had no proof of them.
Defense counsel did have proof of Ogden
Jury asked if Ogden’s stories were in fact true. Trial judge did not allow defense extrinsic evidence of Ogden’s violent nature in. Claimed James had no knowledge of the extrinsic evidence either so it was not relevant, the violent nature of Ogden was only to show the defendant’s state of mind when she gave her daughter of gun, that of fear.
Appellate court holds: It was absolute necessary to her defense for the jury to believe her credibility, that she wasn’t making up stories, and the records proved that what Ernestine James testified had actually taken place.
Because the crux of James’s defense rested on her credibility and documents could corroborate that excluding them was prejudicial and affected the verdict. District Reversed
Problem 1.6: Violin/Gun. Police officer shots guy with violin case, thought it was a gun when he lifted it up. No gun inside, court can rightfully exclude as prejudicial. That said there may be argument to relevance, maybe it would have been heavier with a gun, or it was far less likely the guy was going to aim a violin and that showed the officer was likely lying.
Condition Relevance

Cox v. State (Ind. 1998)
Couple sleeping, POP, and guy shot in the head. Bullet holes found in bedroom window and screen, bullet casing outside the window. Shot fired six inches from window.
Cox suspected when someone says he wanted to kill Leonard (husband). Cox’s friend Hammer was in jail. Hammer was put away for molestation of Leonard’s daughter.
Cox objected to testimony by deputy prosecutor saying that at bond hearing that additional charges were going to be filed and that Hammer’s bond was not reduced. Hammer’s mother was at that hearing.
State could not prove that Cox had heard about that hearing, but the mother of Hammer definitely had and Cox was hanging out with her night in question.
Conditional question on whether Cox had heard about it from mother Hammer.
State introduced evidence that Cox spent almost every day at the Hammer house. Hammer and Cox were close friends and Hammer’s mother attended the hearing. Evidence sufficient to support the inference that Cox had learned what transpired at the hearing.
Conditional relevance generally really applies to almost any chain of inferences if you think about it. It compares the 401 standard (normal) to the 104(b) standard (conditional) for determining relevance of evidence. 104(b) is tougher but still pretty lenient:
Jury could reasonable find the conditional fact … by a preponderance of evidence. Huddleston
Conditionally relevant evidence can only be introduced after you lay groundwork to get it in as well, can’t introduce it out of order.
Prejudicial Evidence
Rule 403 Probativeness v. unfair prejudice. When prejudice substantially outweighs probativeness evidence is unfairly prejudicial.
State v. Bocharski (Ariz. 2001)
Defendant Bocharski had a distinct knife. He lived in the Arizona desert next to crazy old lady. One time he was commented saying he should offer to get rid of her because she was always complaining and praying god would put her out of her misery.
Victim, old lady, was found dead of sixteen stab wounds. Defendant’s knife went missing for three months, there were his fingerprints on victim’s door but they could not be dated. No other evidence was found.
Trial court allowed photos of old ladies corpse. Generally relevant photos are admitted, unless they are unfairly prejudicial.
Of import in this case was that the photos did not go to any contested elements. Defense was not challenging the death of the old lady, or how she died. If they were maybe the photos would be more probative.
Two photos in particular concerned he court, pictures of the defendants hollowed out skull. These were found to have been unfairly prejudicial given their inflammatory nature.
However because the jury did not have an huge reaction to the two photos in question compared to the photos that were relevant (showing wounds and body) it was considered a harmless error.
Do jurors need to be protected from themselves, and outsized reactions to gruesome corpse pictures?
Problem 1.8: Dispute of whether gun automatic or not, government test shows it is, defendant argues it was dirt internally when government tested. Government provides photo of gun being clean along with tons of other guns defendant owns. No picture of internal state of gun however. Seems to be an obviously prejudicial photo, it’s not really relevant to whether it was clean on outside, and the photo of other guns is prejudicial.
Commonwealth v. Serge (Pa. 2006)
Serge shoots wife, prosecution wants to present computer generated animation of how it went down.
Trial court issued jury instruction explaining that the animation was not a definitive recreation of the event, just what the experts though likely had happened.
Also the CGA lacked any real distinguishing details, it was just like a hand drawn explanation of what would have happened really.
Court did not want to be luddite and hold back technology, despite visual appeal of CGA it was not unfairly prejudicial, especially in light of extensive jury instruction telling them not to read too much into the CGA.
United States v. James (Continued)
Dissent: complains that the evidence of Ogden’s violent behavior was unfairly prejudicial on the victim Ogden since even if it was relevant to defendant’s credibility it did not provide enough probative value to overcome the harsh damage it did to the juries view of the victim.
OJ trial, some instances of investigating officer saying nigger is ok, all 41 of them was unfairly prejudicial.
United States v. Myers
Robbery by lone gunman of bank. Myers accused, has a friend who plead to the crime who looks a lot like him.
Defendant objects to evidence introduced about his flight from FBI agents on two occasions subsequent to commission of robbery.
The evidence was let in and jury was instructed to keep it in mind as showing a consciousness
The first flight was reported by Dunn from Florida, who Myers lived with before robbery. She testified FBI tried to get in touch with Myers after robbery and he avoided them. Then when she came to give him clothes in a mall the FBI agents were there and realized she was meeting with Myers. They ran to him without identifying themselves and he ran away.
Afterwards Myers went to PA.
Myers and friend were arrested in Cali. They both tried to flee when they were arrested but just barely, moving only a few feet possibly in shock since they were trapped by cars on all sides while on motor cycle.
More remote in time flight is from incident the greater the likelihood that it resulted from something other than feelings of guilt.
Both incidents of flight were deficient in either physical evidence adducing to flight or flight as it should properly be evidentiarily used. In light of that the two instances being introduced were unfairly prejudicial.
Flight evidence is looked unfavorably by courts since people can leave areas for innocent reasons too. United States v. Jackson (E.D.N.Y. 1975)
People v. Collins (Ca. 1968)
Blond woman and black man rob an old lady.
Prosecution uses statistical evidence to show that the likelihood of it being another interracial couple given what was known about the defendants was one in 12 million.
Court holds that this evidence was unacceptable. The statistician made too many assumptions when determining the odds. Thus the statistic was probably useless. Furthermore it’s incredibly prejudicial for a jury to hear that there is a one in 12 million chance that it could have been another couple who committed the robbery. It encouraged the jurors to rely on an engaging but ultimately irrelevant expert demonstration about who was culpable for the crime.
After Collins math hasn’t disappeared from court but its mainly used in DNA evidence now. Has to be sound, which DNA probabilities are.
United States v. Jackson (E.D.N.Y. 1975)
Defendant, accused of robbing bank, motions to dismiss evidence that he used a false name on being arrested in Georgia shortly after robbery.
Defendant was arrested in Georgia with false ID, and escaped from local jail afterwards.
This is more an issue of evidence of flight, did defendant leave New York to Georgia after robbery. Evidence shows he stayed for months after robbery, and even stayed for a month after indictment.
Also possible that he wasn’t even in Georgia since there is no definitive proof it was him with false id.
Bringing in the Georgia events bring another arrest and crime.
This is unfairly prejudicial when defendant hasn’t been convicted in Georgia and may have had nothing to do with events that took place there. Government evidence entails risk that unrelated crimes will be brought to the attention of the jury. Or the jury might be confused about what exactly they are determining guilt for.
Evidence of arrest in Georgia will be inadmissible at trial provided defendant enters into a stipulation that he was in Georgia shortly after robbery and that while there he used a false name.
This allows the flight inference without bringing in defendant’s arrest and any crimes he may or may not have committed in Georgia.
Old Chief v. United States (1997)
Defendant arrested with gun, violation of law prohibiting possession of firearm by anyone with a prior felony conviction. Lots of defendant’s charged with this crime admit to gun possession after felony conviction to avoid the details of their crime being released.
Defendant made that offer and state spurned it.
Old Chief tried to argue that the name of his prior offense, assault, as contained in record of conviction was irrelevant to prior-conviction element. Court dismisses this argument immediately. Even if other evidentiary proofs were available regarding previous conviction the actual record was fine.
Unfair prejudice of naming the conviction goes against rule allowing prosecution to avoid letting the defendant stipulate elements of a case to paint an incomplete picture of his crime. Here unfair prejudice weighs out because the question is the defendant’s legal status, no other element of the prior conviction is important for the crime in question here. Thus a stipulation of prior felony would have been more than enough for the prosecution, no need to bring in prior felony as assault and prejudice the defendant.
Specialized Relevance
Rule 407: Subsequent Remedies
Rule 408: Compromise
Rule 409: Medical Expenses
Rule 410: Pleas
Rule 411: Liability insurance
Can be admitted for other purposes such as to show feasibility of a subsequent remedial measure for example.
Problem 2.2: Wood chipper case where the company changed up the wood chipper and implied that they didn’t and it was still in use in place where plaintiff’s husband was killed by it as well as at the army. Proper to admit evidence of change to impeach the company, it can’t use 407 to lie.
Rule 407 does not apply to third party repairs.
Williams v. McCoy (N.C. app. 2001)
Personal injury suit from automobile accident. Plaintiff hired attorney after meeting with defendant’s insurance adjuster. Wanted to be able to bring that in because otherwise she looked like a money grubber who immediately hired an attorney even before going to doctor.
Court holds that the prejudice of withholding the testimony regarding the bad experience with the adjustor outweighs the public policy aspirations of rule 411. Defense was using 411 in bad faith and prejudicing plaintiff unfairly.
United States v. Biaggi (2d Cir. 1990)
Defendant in a white collar case argues that it was an error to exclude evidence of immunity negotiations he participated in, since they proved his consciousness of innocence. Government offered him immunity if he could give evidence of wrongdoing for his co-defendants and he declined such a great deal.
Government doesn’t deny immunity but claims it was withdrawn given how little defendant knew. This is the exact same point defendant is trying to make, he didn’t know anything about wrongdoing.
This does not fall under 410, it’s a different character of probativeness between plea bargaining and rejecting immunity because you claim you don’t know anything.
Rule 410 is different from other rules in this family, the evidence it addresses is always barred. Can’t be used to impeach for example. Rule 410 does not protect prosecutors on the other hand by its language. Defendants can bring in evidence of offers to drop charges. But some courts don’t allow this since it would discourage plea bargaining. Also Problem 2.8.
Character Evidence – Propensity
People v. Zackowitz (N.Y. 1930)
Zackowitz kills guy for insulting his wife. Cops to the murder but says he was drunk when he did it, so it wasn’t in the first degree.
State gets in evidence that he had weapons in his apartment.
Unacceptable under propensity rule, the evidence of the guns was used only to establish that the defendant had a murderous and dangerous character.
Character must not be an issue unless defendant chooses to make it one.
Character evidence may be admitted in self defense case to show deceased was murderous and attacker though.
If the question instead had been were pistols brought for this encounter that would have been ok. But here the defendant already owned them for a while. Another ok question would have been was defendant armed with all his weapons prior to the encounter. But here it was pure propensity argument, endeavor to generate atmosphere of professional criminality.
That was unfair.
Dissent disagrees, says that if he been found with the weapons on him that would have been ok. Agrees with prosecution rationale that evidence was brought that defendant had weapons to choose from, and did so to kill.
Rule 404(a) prohibits propensity evidence, which is evidence used to show character predisposed towards criminal behavior, specifically the criminal behavior alleged.
You can get around the propensity box in many ways. 404(b) presents a couple of ways around the propensity box:
Proof of motive, opportunity, intent, preparation, plan, knowledge, identity, absence of mistake or accident.
These around the box presentations of evidence can be excluded when weighed by Rule 403 however.
404(b) is not an exception to 404(a) prohibition, there are real exceptions in 404(a)(1)(2)(3) and 413-415, but 404(b) is simply points out if you get around the box your good.
Problem 3.3: Negligence case, R.R. man has general reputation of drunkenness, why is that ok?
Proof of Modus Operandi is one way around the propensity box. For this route to be legitimate the similarities between two crimes must be so distinctive that the inference that nobody else could have committed this crime overcomes the jury’s temptation to engage in propensity reasoning.
United States v. Trenkler (1st Cir. 1995)
Bomb case. Trenkler charged with creating bomb.
Defendant previously created a bomb, a remote-control radio-activated explosive device. Previously defendant had used various methods unique to his style of bomb building.
The bomb in question was remarkably similar to the previous bomb, it was also remote-control and radio-activated.
Defendant and Prosecution experts clashed over how distinct this bomb design was.
The clincher was a database of bomb incidents which when narrowed down to the characteristics of the bomb at trial had only seven reports.
Court affirmed Trenkler’s conviction under the identity means of around the propensity box. Pointed out bombing is fairly distinct means intimidation and killing. Also the coalescence of all the various similarities the two bombs shared was fairly persuasive. Defense expert conceded that there was a probability that maybe there was a connection between the maker of the two bombs.
Beyond a reasonable doubt is not the conclusion necessary to activate the identity go around.
Not unfairly prejudicial, especially in light of fact that first bomb did not kill or injure anyone and caused little property damage, and trial court minimized risk of unfair prejudice with good jury instruction.

More about evidence

RELEVANCE/Chapter I

a. Probativeness and Materiality (18-30)

i. FRE 401 (20)

1. Definition of “Relevant evidence: evidence having any tendency to make existence of any fact that is of consequence to determination of action [must deal with a material fact] more or less probable than it would be w/out evidence [must be probative of that material fact].

ii. FRE 402 (20)

1. Relevant Evidence Generally Admissible; Irrelevant Evidence Inadmissible. All relevant evidence is admissible, except as otherwise provided by Constitution, by Act of Congress, by these rules, or by other rules prescribed by S.C. pursuant to statutory authority. Evidence which is not relevant is not admissible

a. In order to be relevant under 401, evidence must be probative of a material fact.

iii. Probativeness (20)

1. Must have tendency to make existence of a material fact more/less probable than it would be w/out evidence.

a. This is very, very low standard it is not a preponderance of evidence standard! All that has to happen is that evidence has to make material fact slightly more or less probable than w/out evidence.

2. George F. James, Relevance, Probability and the Law (20)

3. Problem 1.1 Show me the body (21)

4. Problem 1.2 Brotherhood (22)

5. Problem 1.3 Polygraph Consent (22)

iv. Materiality (23)

1. must bear on a fact that is “of consequence” to determination of action

a. EG: If prosecutor in murder case seeks to offer evidence of V’s lost earning potential, this isn’t going to be relevant. However, this same evidence would be relevant in a wrongful death case.

b. NOTE: Materiality depends on the substantive law of JN in question.

2. Problem 1.4 Knowledge (23)

3. Problem 1.5 Voluntary Intoxication (24)

4. United States v. James (25)

a. Relevance of Credibility?

b. Afterthoughts (28)

5. Problem 1.6 Violin Case (30)

b. Conditional Relevance(30-38)

i. Focus on FRE 104(b)

1. Relevancy Conditioned on Fact. When relevancy of evidence depends upon fulfillment of a condition of fact, court shall admit it upon, or subject to, introduction of evidence sufficient to support [jury] finding of condition’s fulfillment.

a. Advisory Notes:

i. In some situations, relevancy of evidence depends upon existence of a particular preliminary fact (“conditional relevancy” ).

1. EG: When spoken statement offered to prove notice to X, it is w/out probative value unless X actually heard it.

2. EG: If letter allegedly from Y offered to establish an admission by Y, it has no probative value unless Y actually wrote it, or authorized it.

b. Rule 104(b) Standard. Huddleston & U.S.: Before admitting evidence under Rule 104(b), the trial judge must determine that, based on the evidence, a reasonable jury could find, by a preponderance of the evidence, that the conditional fact is true.

i. Under Huddleston, proponent doesn’t have to convince judge of this by preponderance. Instead, judge must determine that a jury could find this by preponderance.

ii. In making determination, judge may only consider admissible evidence (so if its hearsay, its not allowed)!!

iii. 104(b) conditional relevancy standard is slightly higher than 401 standard.

iv. 104(b) requires that proponent introduce sufficient evidence that jury could reasonably find conditional fact to be true by preponderance.

ii. Problem 1.7 Threat to Disclose (31)

iii. Bill D’Agostino, Fitzhugh Case: Judge Allows Paternity Motive (31)

iv. Cox v. State (32)

1. Afterthoughts (34)

c. Probativeness v. Risk of Unfair Prejudice: (38)

i. Inflammatory Evidence (38-53)

1. Focus on FRE 403 (38) Exclusion of relevant evidence on grounds of prejudice, confusion, or waste of time.

a. Although relevant, evidence may be excluded if its probative value is substantially outweighed by danger of unfair prejudice, confusion of issues, or misleading jury, or by considerations of undue delay, waste of time, or needless presentation of cumulative evidence.

2. Rule 403 Phrase by Phrase (38)

a. Substantially outweighed by danger of

i. Unfair prejudice

ii. Confusion of issues

iii. Misleading Jury

iv. Considerations of Undue Delay

v. Waste of Time

vi. Needless Presentation of Cumulative Evidence

b. Still a presumption of admittance

c. Appellate Review is for abuse of discretion

d. Last-Ditch Effort objection

e. Judge may admit with limiting instruction to reduce unfair prejudice

f. ACN permits consideration of evidentiary alternatives

3. Photos and Other Inflammatory Evidence (39)

a. State v. Bocharski (39)

i. Relevant photos admissible although they have tendency to prejudice jury against D.

ii. Is photo of nature to incite passion or inflame jury?

iii. Does danger of unfair prejudice substantially outweigh probative value?

b. Problem 1.8 Photo of Guns (43)

c. Andrew Ross Sorkin, This Tyco Videotape Has Been Edited for Content (44)

d. Commonwealth v. Serge (45)

e. Unfair Prejudice = an undue tendency to suggest decision on an improper basis, commonly, though not necessarily, an emotional one (ACN) United States v. James (50)

f. Excerpts from the Ruling on the Fuhrman Tapes (52)

ii. Flight (54-61)

1. Evidence of Flight (54) US v. Myers (54)

a. Flight instruction improper under 403 unless evidence is sufficient to furnish reasonable support for all 4 inferences:

i. From D’s behavior to flight;

1. could not be est. in Myers

ii. From flight to consciousness of guilt;

iii. From consciousness of guilt to consciousness of guilt concerning crime changed; and

1. could not be est in Myers based on fact D had committed another crime in another state

iv. From consciousness of guilt concerning crime charged to actual guilt of crime charged.

b. Afterthoughts (58)

2. Problem 1.9, Fleeing Trouble I

3. Problem 1.10 Fleeing Trouble II

4. Problem 1.10 Staying Put

iii. Probability Evidence (61-75)

1. Probability evidence, particularly in criminal cases, must be critically examined in view of substantial unfairness to D which may result from ill-conceived techniques with which jury is not technically equipped to cope, esp where evidence is based on estimates, validity of which hasn’t been demonstrated. People v. Collins

a. Here it lacked adequate foundation in evidence and statistical theory

b. Distracted jury from proper function of weighing evidence

2. Historical Post Script People v. Collins

iv. Effect of Stipulations on balancing (75-89)

1. US v. Jackson

2. Ct should consider evidentiary alternatives when determining whether to admit evidence under R 403. When only contested issue is D’s convict status, D’s proposed stipulation admitting to this status is sufficient to prevent govt from presenting D’s record under R 403.

a. Generally a stipulation can discount the probative value of a piece of evidence and means there is less chance that the evidence will get in under R 403.

b. Old Chief v. United States Here, the court decides to permit stipulation to prove D’s criminal record over P’s desire to present the record. P claims this destroys the res gestae.

II. Chapter II

a. Specialized Relevance Rules (90-93)

i. Specialized Relevance Rules: Analytical Charts

ii. Specialized Relevance Rules Flowchart

b. Subsequent Remedial Measures (94-107)

i. Focus on FRE 407

1. When, after an injury/harm allegedly caused by an event, measures are taken that, if taken previously, would have made injury/harm less likely to occur, evidence of subsequent measures not admissible to prove negligence, culpable conduct, a defect in a product, a defect in a product’s design, or a need for a warning or instruction.

2. This rule doesn’t require exclusion of evidence of subsequent measures when offered for another purpose, such as proving ownership, control, or feasibility of precautionary measures, if controverted [if other side has disputed this issue], or impeachment.

a. Here admissibility turns on what the evidence is probative of

b. List here is illustrative, non-exhaustive

c. If controverted applies to all exceptions

ii. Problem 2.1, Wolf Attack

iii. Tuer v. Mcdonald-if controverted clause, heart medicine, narrow/broad definition of feasibility

iv. Problem 2.2 Wood Chipper I

v. Problem 2.3 Wood Chipper II

vi. Notes on Rule 407: Subsequent Remdial Measures

c. Compromise Offers and Payment of Medical Expenses (107-116)

i. Focus on FRE 408 and FRE 409

1. FRE 408 Compromise & offers to compromise.

a. (a) Prohibited uses. Evidence of the following is not admissible on behalf of any party, when offered to prove liability for, invalidity of, or amount of a claim that was disputed as to validity or amount, or to impeach through a prior inconsistent statement or contradiction:

i. (1) furnishing or offering or promising to furnish or accepting or offering or promising to accept a valuable consideration in compromising or attempting to compromise the claim ; and

ii. (2) conduct or statements made in compromise negotiations regarding the claim, except when offered in a criminal case and the negotiations related to a claim by a public office or agency in the exercise of regulatory, investigative, or enforcement authority.

b. Permitted uses. This rule does not require exclusion if the evidence is offered for purposes not prohibited by subdivision (a). Examples of permissible purposes include proving a witness’s bias or prejudice ; negating a contention of undue delay; and proving an effort to obstruct a criminal investigation or prosecution.

i. Notes on R 408:

1. Doesn’t require exclusion of any evidence otherwise discoverable merely because it’s presented in course of compromise negotiations.

2. Doesn’t require exclusion when evidence is offered for another purpose, such as proving bias/prejudice of witness, negativing contention of undue delay, or proving an effort to obstruct a criminal investigation or prosecution.

3. Only applies in civil cases.

4. Doesn’t protect offers to compromise before a “claim” of some sort is made

5. Compromise offer inadmissible and as well as conduct/statements made in compromise negotiations

6. ACN policy grounds: irrelevance of this kind of evidence, public policy of compromise and settlement disputes

2. Not absolute ban on all evidence related to settlement negotiations. Rule permits evidence that is offered for any purpose other than est. liability or invalidity of claim. Bankcard America, Inc v. Universal Bancard Systems, Inc. HERE evidence demonstrated good faith belief and state of mind, not liability.

a. Cts will not allow party to use rule as a sword, since otherwise one could trick D into breaching Ks.

b. Minimizing instruction that limited use of word “settlement” to minimalize prejudice.

3. FRE 409 Payment of Medical and Similar Expenses

a. Evidence of furnishing/offering/promising to pay medical, hospital, or similar expenses occasioned by an injury is not admissible to prove liability for the injury.

1. ACN: Relevance: Offers to pay can be product of human impulse & not admission of liability. Policy: Encourages offers to assist.

ii. Problem 2.4, Hotel Inspection

iii. Ray Henry, States Encourage Docs to Apologize

iv. Comparing Rules 408 (Compromise) and Rule 409 (Medical Expenses):

1. Both rules, evidence of offer to pay towards 3d party is not admissible BUT may be admissible to prove bias.

2. Unlike 408, 409 does not protect statements made during offers to pay. Ex. D offers to pay for P’s medical expenses and says “I’m sorry, its all my fault.” While D’s offer can’t be admitted into evidence to prove liability, his statements can.

d. Liability Insurance (116-27)

i. Focus on FRE 411 Liability Insurance

1. Evidence that a person was/was not insured against liability is not admissible upon issue whether person acted negligently or otherwise wrongfully. This rule doesn’t require exclusion of liability insurance evidence when offered for another purpose, such as proof of agency, ownership, or control, or bias or prejudice of a witness [this is a non-exclusive list of exceptions].

2. ACN: Relevance: At best, inference of fault from fact of insurance coverage is tenuous, as is its converse. Policy: Knowledge of presence/absence of liability insurance might induce juries to decide cases on improper grounds.

a. Rule broad and includes contributory negligence or other fault of P as well as fault of D.

ii. R. 411 does not absolutely bar admission of evidence concerning liability insurance. It only excludes this evidence solely on issue of negligent or wrongful conduct but not if it is offered for another purpose.

1. In reviewing whether or not to admit evidence of liability insurance under 411, court must consider Rule 403 balancing test.

2. Williams v. McCoyP was not trying to admit evidence to prove negligence/wrongful conduct, but instead to show why she hired an attorney before seeing a dr. so its admissible under R 411.

iii. Problem 2.5 Claims Adjuster

iv. Problem 2.6 Malpractice Insurer

v. Problem 2.7 Failure to Report

vi. Shari Seidman Diamond and Neil Vidmar, Jury Room Ruminations on Forbidden Toopics

e. Pleas in Criminal Cases (127-33)

i. Focus on FRE 410

1. Inadmissibility of pleas, offers to plea, plea discussions & related statements. Except as otherwise provided in this rule, evidence of following is not, in any civil or criminal proceeding, admissible against D who made plea or was a participant in plea discussions:

a. (1) a plea of guilty which was later withdrawn;

b. (2) a plea of nolo contendere;

c. (3) any statement made in the course of any proceeding under Rule 11 of the FR of criminal procedure or comparable state procedure regarding either of the foregoing pleas; or

d. (4) any statement made in course of plea discussions with an attorney for prosecuting authority which do not result in a plea of guilty or which result in a plea of guilty later withdrawn.

2. However, such a statement is admissible (i) in any proceeding wherein another statement made in course of same plea/plea discussions has been introduced & statement ought in fairness be considered contemporaneously with it, or (ii) in a criminal proceeding for perjury or false statement if statement was made by D under oath, on the record & in the presence Of Counsel .

a. Rationale: Relevance: Plea not necessarily an indication of guilt D may plea if he realizes that he has the weaker position. Policy: Encouarge pleas, criminal justice system would break down if every case went to trial.

b. Notes

i. Like 408, 410 bars evidence of actual plea negotiations & statements made during these negotiations. Statements made by D during plea negotiations are not admissible to impeach her should she later testify differently at trial. Bars evidence of pleas & plea discussions only when offered “against D.” By its terms, rule doesn’t prevent D from presenting evidence that govt. offered to drop charge during plea discussions.

ii. Uniquely, barred regardless of the purpose of admitting it.

ii. Problems 2.8 Prosecutor’s Offer

iii. Due to policy reasons behind 410 (encouraging plea negotiations), some cts. have ignored literal language of rule & expanded it to bar evidence used against both D & govt. (U.S. v. Biaggi).

iv. 410 does not bar defense evidence of government offered immunity; the Rule only applies to plea negotiations/discussions. (Biaggi). United States v. Biaggi

v. Problem 2.9 Mariotta’s Retrial

vi. Notes on Rule 410: Pleas, Plea Discussions, and Related Statements

f. Specialized Relevance Rules Afterthoughts (133-34)

III. Chapter III

a. The Character-Propensity Prohibition (135-51)

i. Focus on FRE 404 Character evidence not admissible to prove conduct; exceptions; other crimes.

1. (a) Character evidence generally. Evidence of a person’s character/trait of character is not admissible for purpose of proving action in conformity therewith on a particular occasion; except:

a. (1) Character of accused. Evidence of a pertinent trait of character offered by the accused, or by prosecution to rebut the same, or if evidence of a trait of character of AV is offered by accused and admitted under 404(a)(2), evidence of the same trait of character of the accused offered by the prosecution;

b. (2) Character of alleged victim. Evidence of a pertinent trait of character of AV offered by accused, or by prosecution to rebut the same, or evidence of a character trait of peacefulness of AV offered prosecution in a homicide case to rebut evidence that AV was first aggressor;

c. (3) Character of witness. Evidence of character of a witness, as provided in 607, 608 & 609.

2. (b) Other crimes, wrongs, or acts. Evidence of other crimes, wrongs, or acts is not admissible to prove character of a person in order to show action in conformity therewith. It may, however, be admissible for other purposes, such as

a. proof of motive

b. opportunity

c. intent

d. preparation

e. plan

f. knowledge

g. identity

h. or absence of mistake or accident

i. provided that upon request by accused prosecution in a criminal case shall provide reasonable notice in advance of trial, or during trial if court excuses pretrial notice on good cause shown, of general nature of any such evidence it intends to introduce at trial (NOTE: 404(b) covers all acts, not necessarily just “bad” acts and, further, these acts don’t have to be “prior” acts; 404(b) covers any act, whether prior, concurrent or subsequent to act charged).

a. ACN: 404 only bars character evidence if offered to prove action in conformity therewith other purposes are fine. If character is itself an element of the crime, claim or defense. Here, 404 does not apply & character evidence is admissible.

i. Once admissibility of character evidence in some form is established under this rule, go to 405 to determine appropriate method of proof.

More evidence

Introduction: Tanner & FRE 606(b)

· Tanner v. United States, 483 U.S. 107 (1987)

o FACTS: The one where the jurors were drinking and blowing lines during trial. A juror came out after the verdict and told the defense attorney what happened.

o ISSUE: Should the jury verdict be impeached/

o HOLDING: NO, verdict stands.

§ C/L prohibits the admission of juror testimony to impeach a jury verdict.

§ Turns on the nature of the allegation: whether it is internal or external. Jury verdicts cannot be reversed if they are internal matters, but they may be overturned on external matters.

§ Does not turn on where this takes place, but what outside influences affect the internal workings of the jury.

· D’s right to a competent jury

· What evidence is admissible to prove a claim?

o Under 606(b): any testimony from a non-juror

o Difference that matters: internal vs. external matters.

· Video Clips: documentary/film

o William Ward: deceased brother

o Delbert Ward: D charged with William’s death

o Roscoe & Lemon Ward: other brothers

§ PROSECUTION: confession + autopsy:

· Try to make inadmissible will come in though. Lessen the autopsy by impeachment.

o à Police had motive with making money off of land. Can’t talk to neighbor (hearsay)

· Put D on the stand

· Take evidence that points toward motive.

§ DEFENSE:

· Impeach confession

· Testimony to character (à FRE 602)

o No good, must have personal knowledge

· Neighbor who shows up:

o Relevance

o Evidence of character (FRE 404) is not admissible w/o there being an exception.

GENERAL PRINCIPLES OF RELEVANCE

Probativeness Versus the Risk of Unfair Prejudice

Rule 403: EXCLUSION OF RELEVANT EVIDENCE ON GROUNDS OF PREJUDICE, CONFUSION OR WASTE OF TIME

Although relevant, evidence may be excluded if its probative value is substantially outweighed by the danger of unfair prejudice, confusion of the issues, or misleading the jury or by considerations of undue delay, waste of time or needless presentation of cumulative evidence.

o ACN: “consideration should be given to the probable effectiveness or lack of effectiveness of limiting instructions. The availability of other means of proof may also be an appropriate factor,”

o FRE 403, word-by-word:

§ Evidence may be excluded: this is exercised under the trial judge’s discretion and are reviewable on appeal only for abuse of discretion.

§ If its probative value is substantially outweighed by: liberal evidence rule. Only if the listed evils “substantially outweigh” the probative value does FRE 403 give the judge discretion to exclude the evidence.

§ The danger of unfair prejudice: All relevant evidence is inherently prejudicial, but only unfair prejudice, that substantially outweighs the probative value, permits the exclusion of a relevant matter.

§ The danger of confusion of the issues, or misleading the jury: distracting the jury is grounds for excluding evidence.

· Even sheer waste of time may justify exclusion.

· “Any tendency” = probative

· “Of consequence” = materiality

o Photos and Other Inflammatory Evidence

§ State v. Bocharski (AZ 2001)

· FACTS: An old lady was stabbed 16 times in the head. The ? did not dispute how she died, but only disputed identity. ? proffered pictures of her decompensating body.

· ISSUE: Should the pictures of her skull been excluded?

o Concurrence: The issue is simply whether the trial CT abused its discretion in weighing PV against the prejudicial effect under FRE 403?

· HOLDING: YES they should have been excluded.

o If a photo “is of a nature to incite passion or inflame the jury” the CT must determine whether the danger of unfair prejudice substantially outweighs the exhibit’s probative value. Trial CT decision will not be disturbed unless there is a clear abuse of discretion.

o In murder cases, pictures are typically facts of consequence. But if the ? does not dispute the fact of consequence (how the victim died), then the pictures probative value is minimal.

o The pictures may have proved something, but they have minimal tendency to prove or disprove identity (the only fact at issue).

o Even though the judge did do a 403-balancing test, he ruled incorrectly.

o Gory photos: the unfair part is where the jury is inflamed. It may over-persuade; emotional response may give too much weight than its actual PV.

o Cause of death not disputed; type of weapon not disputed; or that the murderer intended to kill

o * The availability of other evidence speaks to the strength, fairness and PV of this piece of evidence.

o * Just because ? doesn’t proffer other evidence, doesn’t mean the ? doesn’t still have burden of proof.

o Exhibits 46 & 47 is harmless error: because the jury didn’t go all the way with the charges

o To mitigate damage:

§ Cross examine expert; cross autopsy; character witness

§ Limiting instruction à Serge. “Nothing going to identity.”

§ Motion for B&W pictures; motion for small pictures

§ Potential for secondary trauma à not good for ?

§ Commonwealth v. Serge (PA 2006)

· FACTS: The case where the prosecution created an animation as a fair and accurate depiction of expert reconstructive testimony and to exclude any inflammatory features that may cause unfair prejudice. The CGA case.

· ISSUE: Whether the admission of the CGA was proper.

· HOLDING: The CGA is admissible.

o It’s admissible if it is:

§ (1) A fair and accurate representation of the evidence

§ (2) Is relevant under FRE 401 and 402, and

§ (3) The PV is not outweighed by danger of unfair prejudice pursuant to FRE 403.

· The fact that the prosecution had a higher budget is a consideration, but it is balanced by limiting instructions to the jury.

o The CG met all three of these criteria, therefore its admissible.

o ?: Unfair prejudice (people respond more to video); $$$ considerations

o ?: CG explains the evidence particularly well, not gruesome at all, no noise even.

§ United States v. James (9th Cir. 1999)

· FACTS: The one where the woman was dating a dangerous ?, who told her various violent stories. She was with her daughter and feared the ? was going to hurt them. She handed her daughter a gun and shot him.

· ISSUE: Whether evidence that the ? really had a criminal record is admissible

· HOLDING: Criminal record is not admissible.

o The mother did not have knowledge of the records at the time, so it has no bearing on her state of mind.

o The only relevant factor is the mother’s state of mind for a self-defense claim.

o Though it goes to show that she is more credible if the papers were admitted, the papers bring unfair prejudice to the ? in balancing it with its minimal PV on a fact of consequence.

· Evidence ? puts forth: criminal record of victim. 3 Instances.

· ?’s claim: self defense: she must have

o (1) Reasonable belief &

o (2) Honest belief of death or serious bodily injury.

· ?’s argument: a criminal record she’s never seen is immaterial

o EF: Victim’s criminal record

o INF: Likely he told her if its true

o INF: Goes to D’s credibility

o INF: Then she’s probably telling the truth about her fear of ?. Goes to R-ness.

· DISSENT:

o MAJ: CT erred when it prevented evidence that victim in fact committed crime w/ FRE 403.

o DISSENT:

§ Step 1: Relevancy (401)

§ Step 2: Unfair Prejudice (403)

§ Step 3: PV (403)

· Prejudice the victim as a bad man. Concerned that the jury will decide he’s a bad man regardless of the evidence.

· Issue: whether the PV is sub. Outweighed. But this depends on the PV. Is a piece of evidence probative in isolation or probative in the context of other evidence?

· Advisory committee: the availability of other evidence is a factor. In fact, you don’t look at evidence in isolation.

§ Fushman Tapes & OJ Simpson:

· Chain of inference:

o EF: Discriminatory language

o INF: Bias

o FOC: Not credible, impeach testimony.

· Unfair prejudice = piling on evidence. The cumulative quality.

o EVIDENCE OF FLIGHT

§ United States v. Myers (5th Cir. 1977)

· FACTS: the one where the many is running around robbing banks and running away from police. Evidence of flight was admitted.

· ISSUE: Whether evidence of ?’s flight is admissible?

· HOLDING: It was in error to admit the evidence of flight. REVERSED.

o The PV of flight depends on the degree of confidence in this chain on inferences:

§ (1) From ?’s behavior to flight

§ (2) From flight to consciousness of guilt

§ (3) From consciousness of guilt to consciousness of guilt concerning the crime charged.

§ (4) From consciousness of guilt concerning the crime charged to actual guilt of the crime charged.

o W/o knowing whether the ? committed the FL robbery, it is impossible to say whether the CA flight resulted from feelings of guilt, and thus from feelings of actual guilt.

o “Because of the inherent unreliability of evidence of flight, and the danger of prejudice its use may entail, a flight instruction is improper unless the evidence is sufficient to furnish reasonable support for all four of the necessary inferences.”

§ Because people may leave a jurisdiction for a number of reasons, CTs are reluctant to admit evidence of flight. United States v. Jackson.

· CLASS NOTES

o Delbert Example: Chain of inferences:

§ #1 : Brother’s testimony

· EF: No sleep, brother would wake him up, wet his bed. à

· INF: Delbert is bothered by his brother, annoyed. à

· FOC: Delbert had a motive to kill his brother.

o Motive to kill is an element of a murder, and it has some tendency to show mens rea.

§ #2:

· EF: William told ? he would die and a brother would be arrested. à

· INF: Prosecution: an unlikely set of circumstances. To challenge ?’s credibility à

· FOC: He’s lying about everything, including the murder.

§ Problem 1.1 (pg. 21)

· EF: Wife’s reaction: “Where’s the body?” unusual response

· INF: She has information.

· INF Her husband told her.

· FOC: She has some involvement in the murder.

· But there are notions of human behavior in play and it goes to jury’s weighing of credibility.

· Even if jury accepts all of these inferences, it’s not enough for the charge. “A brick doesn’t make a wall.”

o MATERIALITY

§ Problem 1.4:

· Elements of charge: (1) transport firearms, (2) convicted up to one year (this is for strict liability, no intent necessary)

· Thus, the fact that the D didn’t know he could be put in jail for a year is irrelevant. No FOC.

§ Problem 1.5

· Voluntary intoxication ? admissible. à M.R.

o Purposely/knowingly intent to ill: if this evidence were admissible, it would (1) shift the burden of proof to prosecution and (2) redefine murder.

o But voluntary intoxication could be admissible for other purposes.

§ Problem 1.6: Violin case, another self-defense.

§ Problem 1.8 (pg. 43): ? shows gun-shooting 2x. ?’s expert says the gun is dirty. ? presents pic of the gun clean on the outside. ? says it’s relevant but not probative.

o Cox v. State:

§ Conditional relevance: ? knew what happened at the previous hearing.

· EF: Hearing, felonies denied bond.

o (1) ?’s friend Hammer and he’s in Hammer’s house with mother

o (2) Hammer’s mom told ? about the hearing.

· INF: ? becomes angry, vengeful, witness tampering.

· INF: Motive to kill

· FOC: has tendency to show elements of a murder.

§ The CT in determining admissibility, reviews by preponderance of the evidence.

§ Reasonable jury could find by preponderance of evidence by certain facts. Low standard.

o Fitzhugh Case Page 31.

o Probability evidence

o People v. Collins (Supreme CT CA 1968)

§ ISSUE: Whether evidence of mathematical probability has been properly introduced and used by the prosecution in a criminal case.

§ HOLDING: NO it infected the role of the jury to determine guilt or innocence.

· Two errors: (1) expert testimony lacked adequate foundation in evidence and statistical theory: assumed that the ? in fact possessed characteristics and that they were the only couple with those characteristics. (2) the matter distracted the jury from weighing the evidence, and encouraged the jury to rely solely on math (inadequate proof of statistical independence)

§ Legacy: DNA evidence is fundamentally probabilistic. Police can contaminate samples in the lab, but such risks of human fault to not justify excluded DNA tests results because jurors can understand the risks and assess them.

§ Nothing in formula that witness are correct that links the D to the crime. The math left a gap.

§ Didn’t provide for foundation of D’s math

§ Each factor has to be independent but a lot of them are not.

§ A piece of evidence can be prejudicial. The ? could easily tear this info apart. Not a case about bad defense: the CT must be the gatekeeper of the trial.

· Effect of stipulations:

o Old Chief v. US 519 U.S. 172 (1997)

§ ISSUE: whether admitting an entire record of a prior judgment, when only an element of the conviction is necessary, is an abuse of discretion.

§ HODLING: YES, it’s an abuse of discretion.

· If the ? stipulated the prior conviction, then the ruling should be made on the basis of unfair prejudice or cumulative evidence.

o “unfair prejudice” = an undue tendency to suggest decision on an improper basis, commonly, though not necessarily, an emotional one.” FRE 403 CAN.

· Probative value may be calculated based on evidentiary alternatives. Its 401 relevance is not affected.

· “When FRE 403 confers discretion by providing that evidence may be excluded, the discretionary judgment may be informed not only by assessing an evidentiary item’s twin tendencies, but by placing the result of that assessment alongside similar assessments of evidentiary alternatives.”

· But this does not mean that a ? may admit his way out of a full evidentiary hearing. But details may constitute waste of time. The most the jury needs to know is that the conviction admitted by the ? falls within the class of crimes that Congress thought should bar a convict from possession a gun, and this point may be made readily in the ?’s admission and underscored in the court’s jury instructions.”

Conclusion

Notes

See Also

About the Author/s and Rewiever/s

Author: admin

References and Further Reading

About the Author/s and Reviewer/s

Author: admin

Mentioned in these Entries

Attorney, Court reporter, Of Counsel.


Posted

in

, , ,

by

Comments

Leave a Reply

Your email address will not be published. Required fields are marked *